Sei sulla pagina 1di 128
Sub - Junior Level (Middle School Classes) SAMPLE QUESTIONS AND SOLUTIONS for Mathematics Talent Tests Sub-Junior Level (Middle School Classes) R. Athmaraman R. Vijayalakshmi S.R Santhanam THE ASSOCATION OF MATHEMATICS TEACHERS OF INDIA B-19, Vijay Avenue, 85/37, Venkatarangam Street, Triplicane, Chennai - 600 005. Tel: (044)-28441523 E-mail : amti@vsnl.com Web site: amtionline.com BETWEEN US Dear reader, The book, a reference book for Sub-Juniors, published in 2002 had to be reprinted again in 2004 which also has been fully sold out. Hence it was felt a revised and enlarged edition might be brought out which is now in your hands. The AMTI has been conducting the National Talents Examinations in Mathematics for the past 37 years and due to popular demand it has been decided to introduce the pattern from class 4 and this book may be referred even by them though it is primarily meant for children of classes 7 & 8. Sri R.Athmaraman, our senior member, took up this work seriously ably assisted by Kumari R.Vijayalakshmi and Sri S.R.Santhanam to bring out the first edition. I take this opportunity to express my grateful thanks to them. When we decided to bring out the third revised edition after the reprint of 2™ one, Sri.R.Athmaraman, took the initiative to bring this one in record time. He deserves our deep appreciation for the same. We hope and trust that the children will get the necessary lead they need from this book and increase the population of talented in our society. Looking forward to your encouraging response and feed back, and with kind regards / best wishes, Yours sincerely, (MMAHADEVAN) ABOUT THIS BOOK Problem solving is a component of everyday activity. It requires analytical thinking and certainly does not use the concepts taught in the schools and colleges in a direct way. In particular the mathematics that is taught in our schools is mostly devoid of any direct use except on rare occasions. Do you compute compound interest using the formula when you go to a bank? Do you have only specified quadrilaterals and the like when you plan to purchase a plot of land? Do you solve simultaneous equations when you have a problem of two variables in life situation? In short you do not confront routine problems as and when you see them. Most problems are non-routine and it is equally true that these non-routine (shall we call them unusual?) problems need only routine tools for getting solved! Still we do not initiate non- routine problems in our regular classes, leave alone enrichment and club activities. The propagators of non-routine problem solving strongly believe that as an individual, one should get acquainted with tools of solving problems and concurrently know how to manipulate them and apply them in an altogether unfamiliar situation. The mentors of the AMTI are one such group. They strongly advocate familiarizing the techniques of problem solving to students at all stages. That is why this booklet has been thought of for publication, even when the reception by teachers and students for Talent tests at the Junior and Inter levels is marked by reluctance and a bit of uneasiness and (math) anxiety. One will find in this book, samples of questions of objective and traditional types, which require divergent thinking to solve them. The reader will agree that there are many more (and perhaps simpler) ways of solving them. The problems given here are meant to show that even the ones that look ‘troublesome’ have elementary solutions and also a few ‘simple’ problems demand careful analysis and application of tools. It is hoped that the collection will give a ‘feel’ of a general nature of questions asked in Talent tests at a very elementary level and illustrate why such problems are called ‘non-routine’. To solve these questions, only the very basic knowledge of mathematics at the VIII standard level is all that is required. The book needs only-minimal requisites for understanding and use. The AMTI hopes that the teachers and students, after going through the book, will understand that most non-routine problems are more accessible than they imagine. R.ATHMARAMAN Editor CONTENTS Chapter Page Between Us ..seeeesevee atelerate efe.oid a hiwierotos fe © AT About this book .....sseeereeeeeeee sceccessceee 1. Basic Number Operations .......- aiate 89 sseeel 2. Pattern Detection ......seseseeeeereeeeeeeee 27 3. Ratio And Percentage .........s.0es000+ or) 4, Clocks, Time And Distance ..........- one AZ 5. Indices ........++ et eeeeeeeeeees seen ween 48 6. Algebra .........5- ibletaasie oe sa oi res) 7 Geometry ......555 Deesine diene tie altciarteats -70 8. Logical Arguments ........... ce eeeeeeeeees L00 9. Number Theory . ...110 10. Data Handling .. CHAPTER - I BASIC NUMBER OPERATIONS PART-I 1, The number of non negative integers which are less than 1000 and end with only one zero is (A) 90 (B) 99 (C) 91 (D) 100 Screening Test 2004 Solution: Required type of numbers will be nine in each of the ten subsets {1,2,---99}, {100,101,---199}--- {900,9012,---999}. This accounts for 90. Including 0 we have, therefore, 91 such numbers. «. Answer is (C) 2. The numbers 52, 32, 12, 43, 25, 18, 56, 36, 16, 50 are grouped in pairs in such a way that the sum of each pair is the same. ‘The number paired with 18 is A) 52 B) 50 ©) 43 D) 36 Screening Test 2002 Solution: Arrange the numbers in ascending order and group them in 12 16 18 ERS 50 52 56 pairs as shown below: This enables us to get equal sums of pairs: (12, 56), (16, 52), (18, 50), (25, 43), and (32, 36). . Answer is (B). 3. A student is ranked 9'* from the top and 38" from the bottom in a class. The number of students in the class is A) 45 B) 46 c) 47 D) 48 Screening Test 2004 2 Mathematics Talent Tests - Sub-Junior Level Solution: oth from top Bottom ‘Top 7 gaps ‘Sgaps 38th from bottom .. There are (37 +8 =)45 gaps between the topmost and the bottommost students. i.e., there are 46 students in the class. . Answer is (B). Aliter: ‘There are 37 students below and 8 students above him. Thus on the whole there are 37+1+8=46 students in the class. . Answer is (B). 4, In a queue, Amar is 10‘ from the front while Akbar is 25th from behind and Antony is just in the middle of the two. If there be 50 persons in the queue, then the position occupied by Antony from the front is A) 16% B) 18t* C) 19% D) 30% Antony e Amar J Akbar iy 10th 25th from x from y / 50, Between Amar and Akbar there are 15 people. Antony must be in the 8th position from Amar. :. Answer is (B). ‘The digits of the year 2000 add up to 2, In how many years has this happened since the year 1 till this year 2004? (A) 3 (B) 6 (C)9 (D) 10 Screening Test 2004 s x Sample Questions and Solutions Solution: Number of single digited numbers with sum two = 1 Number of two digited numbers with sum two = Number of three digited numbers with sum two Number of four digited numbers with sum two = 4 Hence total such numbers with sum two = 1+2+3+4=10 These ten numbérs are 2, 11, 20, 101, 110, 200, 1001, 1010, 1100 and 2000; ~. Answer is (D) The sum of seven consecutive natural numbers is 84. The difference between the largest and the smallest among them is A)T7 B) 6 c)14 D) 21 Screening Test 2003 Whatever be the set of seven consecutive natural numbers, the seventh natural number is always 6 more than the first natural number. .. Difference between the largest and the smallest is 6. [Note: 84 is divisible by 7. .-, It can represent the sum of 7 consecutive natural numbers. This is because of the property by ‘n’ consecutive natural numbers is divisible by ‘n’, if‘n’ is odd. Check] . Answer is (B) . If the sum of two consecutive odd numbers is 2004, then the smaller of the two numbers could be A) 2001 B) 1001 C) 1003 D)1 . Screening Test 2003 Solution: Let: the two consecutive odd numbers be z — 1 and +1. Given c-1l+ar+1 = 2004 2x 2004 1002. 4 Mathematics Talent Tests - Sub-Junior Level +. The smallest number is z — 1 = 1001. Answer is (B) 8, The'sum of nine consecutive whole numbers is 99. The largest of these integers is A) 19 B) 15 o)17 D) 21 Screening Test 2002 Solution: Let the consecutive whole numbers be 01,42, 03, 4,05, 5,47,a3 and ag. Their sum = 99. «. Their average as = = 11. Since as is 11, ay = 15. <. Answer is (B). Aliter: Let the numbers be a,a+1,---,a+8 so that their sum is 9a + 842 = 90+ 36 = 99, ©. 9a = 63 or a=7. .. The largest of these numbers is a+8 = 15. ©. Answer is (B). © . The sum of the digits of the number 10” — 1 is 3798. The value of n is (A) 431 (B) 673 (©) 422 (D) 501 Screening Test 2004 Solution: 10"—1 is an n digited number with all digits equal to 9, Hence sum of its digits is 9n. But it is given as 3798. n= 422. -. Answer is (C) 10. In the following display each letter represents a digit. 3[B[c[D[=#/8[G/H|r If the sum of any three successive digits is 18, find the value of H. Final Test 2002 Sample Questions and Solutions Solution: 3+ B+C=18 B+C+D=18 D+E+8=18 E+8+G=18 8+G+H=18 11. The adjacent number pyramid is formed by filling up each unshaded square with the sum of the two numbers connected to it from the row below. (Thus 5 is obtained by adding 3 and 2). > = > > > B+C=15 15+ D=18. «. D=3 U+B=18 -, E=7 15+G=18 1. G=3 U+H=18. .. H=7 The top number obtained at the end is 45. The value of X is A) 6 B) 9 Solution: From the figure, C = 7+2, D=2+3. B= 5+D=2+8. C+D=104+22 o A+B = 30418 =45 (given). _ 45-18 | easy <. Answer is (B). 9. ©) 17 D) 12 Screening Test 2002 12. The units digit of the sum of the first 99 whole numbers (that is, OF 14+2+---+98) is A)O B)1 os D)9 Screening Test 2003 6 Mathematics Talent Tests ~ Sub-Junior Level Solution: OF1L4+ 2434445464748 4+9+++- +96 +97 +98 = [(0+10+20+30+-++ +90] +((1 +9) + (2+8) + (347) + (446) + (11 + 19) +(12 + 18) + (13 + 17) + (14+ 16) +--+ +(81 + 89) + (82 + 88) + (83 + 87):-+ (84 + 86)] +((92 + 98) + (93 + 97) +.(94 + 96)] + (5 +15) + (25 + 35) + (45 + 55) + (65 + 75) + (85 + 95) + 91. ‘The sum ends with 1 since the grouped terms other than 91 yields a sum ending with 0. Aliter: OFL424+344454---+964+974+98 = 142434445 4+--4+96 +97 +98 = (1498) + (2+97) + (3 +96) +--+ + (49 + 50) = 49 pairs of 99 = 49x99 which ends in'1. (or) The units place of the result of 49 x 99 is 1. Aliter: OF1F243444-5-498 (14243+4+-+-+97+4 98 +99) — 99 (1 +99) + (2 + 98) + (3 +97) + (4496) ++°* +(49 +51) +50 — 99 = #* 0-9 =e #1 11, The last digit is 1. ©. Answer is (B) Sample Questions and Solutions 7 13. How many times should 2002 be subtracted from 112113 to get remainder 2003? A) 53 B) 55 C) 65 D) 68 Required no. of times = (12135-2009) — 55 ~. Answer is (B) 14. If 8 is subtracted from twice the cube of 5, then the result is not divisible by A) 121 B) ll C) 2 D)3 Screening Test 2002 Solution: Cube of 5 is 125, Twice this cube is 250. when 8 is subtracted from this, we get 242. Since 242 = 11 x 11 x 2, it is not divisible by 3. ©. Answer is (D). 1/D|E 30| F |G H|20| kK M |N | 10 15. Here is a magic square, created using the four numbers 10, 20, 30, 40 once in each row, column or diagonal. The value of A+B is A) 10 B) 20 C) 30D) 40 Solution: C must be 40, \ the only number left along the diagonal. Cc ©. Magic total is 30 40 + 30 + 30 + 10 = 100. Consider the column starting with 10. H has to be 20 or 40. But H cannot be 20 since it is already in the row B[H{[20[K i rlol>|o 20 8 Mathematics Talent Tests - Sub-Junior Level J. H=40 and M = 20. 10 L+H+F+E = 100. 30 E cannot be 10 or 20 since it B|H|20|k is in the bottommost row. M L cannot be 40 since it is in the first column, .. L has to be 30. <. From the first columa, 40+ A+B+30 100. * A+B = 30. cle] >|a . Answer is (C). Aliter: Clearly A+B has to be greater than or equal to 30. The two options possible are 30 and 40. From the second row A cannot ibe 30 and from the third row B cannot be 20. Therefore, A has to be 20 and B has to be 10 with A+B=30. 16. In the addition problem given, letters awPr awe A,B,C stand for distinct digits. The values +. of A,B,C are respectively, + A} 1,98 B) 18,9 A BC C) 1,79 D) 1,9,7 Screening Test 2002 Solution: No number starts with zero and the sum of 3 single digit numbers has a maximum carry of 2. => A can be 1 or 2. From the options given, A is clearly 1. From the addition of ‘units’ column A+B+C=10+C. “ B=% Sample Questions and Solutions 9 From the addition of ‘tens’ column. 1 (carry) +A+B+C=10A+B or 1+1+9+C=10+9 or C=8 A=1, B=9, . Answer is (A). 17. If a, b, c are consecutive numbers in the increasing order such that abe + c¢ b a_ then a cannot be equal to Zao A)1 B) 2 ©)3 D)4 The following are the solutions to the addition problem. 123 234 345 +3 21lo+ 4320+ 543 444 6 6 6 Smeme Therefore a 1 a 2 a = 3 b = 2 b = 3 b= 4 or or c = 3 c 4 ec = 5 (ae d = 6 d = 8 Thus a #4.” ., Answer is (D). 18. The number of whole numbers between V8 and 80 is A)5 B) 6 o)7 D)8 Since we are concerned with whole numbers only, it is enough if we find those lying between V9 (ie. 3) and V1 (ie. 9). ‘These are 3,4,5,6,7 and 8. 2. Answer is (B). 10 Mathematics Talent Tests - Sub-Junior Level 19, In the multiplication shown, D is a single digit. The value 7D x D2 6396 of D should be A)8 B)6 c)9 D)3 Here, D x2 gives a number ending in 6. Therefore, D must be 3 or 8. Here, the product is bigger than 6000. Therefore, B must be 8 only. .. Answer is (A). 20. The value of the product (2 — (4) x (2 = (8), (2— (§)) x «(2 = (588) A)l B) xaos ©) F003 D) 8 Product = $x $x $x +++ x Ros 2005 ere is . Answer is (D). 21. In the given multiplication, @ and 6 are natural numbers. Then a+6 is 3a A)5 B)6 C)8 D)9 ue Screening Test 2003 70 ae 140 olution: Ta70 From 3ax2 = 70, we get a= 5 Now, Jaxb = 140 (or) 35xb = 140 (or) b= = 4ada4 a+b=6+4=9 :, Answer is (D) Sample Questions and Solutions u 22. I multiplied a natural number by 18 and another natural number by 21. Then I added the products. Which one of the following could be the sum of the two products? A) 2002 B) 2003 C) 2004 D) 2005 Screening Test 2003 Solution: 18 and 21 are multiples of 3. Therefore the natural numbers multiplied by 18, 21 are also multiples of 3. Their sum should also be a multiple of 3. But 2004 is the only option which is a multiple of 3. . Answer is (C) 23. Ram divides a number by 1209 and gets a remainder 62. If Shyam divides the same number by 31, then what is the remainder? A)3 B)7 oo D)4 Number = (1209 x Quotient) + 62 = (31 x 39 x Quotient) +(31 x 2)which is divisible by 31. Therefore, Shyam’s remainder = 0. .. Answer is (C). 24. A transport company’s vans each carry a maximum load of 12 tonnes. 24 sealed boxes each weighing 5 tonnes have to be transported to a factory. The number of van loads needed to do this is (A) 9 (B) 10 (0) 11 (D) 12 Screening Test 2004 Solution: Two sealed boxes weigh 10 tonnes and three sealed boxes weigh 15 tonnes. Thus the van can carry a maximum of two sealed boxes. Hence number of van loads needed to transport 24 sealed boxes is 3 = 12. . Answer is (D) 12 Mathematics Talent Tests - Sub-Junior Level 25. Five of the six numbers 3,5,7,9,11 and 13 when multiplied gives 19305. The sixth number that is not needed among the given numbers is A)3 B)7 oyu D)13 149434045 =18. Therefore, 19305 is divisible by 3 and 9. It is also divisible by 5, 11 and 13. :. Answer is (B). 26. Given 1z6y7 is a five-digit number divisible by 9. The number of ordered pairs (x,y) satisfying this is A)5 B)6 08 D) 11 By divisibility test for 9, (since x,y are single digits) l+2+6+y+7 = 18 or 27 or 36 or --- + i+2+y = 180r27 or 36--- = tty = 4orl3or22--- But z,y being single digits c+ y cannot be 22--- 2s. ety =4or 13. <. (ty) are (0,4), (1,3), (2,2), (3, 1), (4,0), or (4,9), (5, 8)(6, 7), (7,6), (8, 5); (9,4). ‘Thus there are 11 ordered pairs satisfying the condition. <. Answer is (D) 27. The GCD and LCM of two numbers a,b are respectively 27 and 2079. If a is divided by 9, the quotient is 21. Then b is A) 243 B) 297 C) 189 D) 1 a=21x9= 189 y= (GOD x LCM) _ (27 x 2079) <= 297 ‘Therefore, ow a «. Answer is (B). Sample Questions and Solutions 13 28. If 0a=18. ‘Thus, 9(230 +z)? = 492804, and hence (230-+2)? = 54756. Taking roots on both sides, 230 + x = 54756 = 234. So, 2 =234—230=4. . at+e=8+4=12. ~. Answer is (C). 30. 146 is a natural number whose “digit-product” is 1x4x6 = 24. How many such three-digit numbers are possible with “digit- product” 24? A)4 B) 18 ©) 21 D) 16 Screening Test 2003 Solution: The factors of 24 are 1,2,3,4,6,8,12 and 24 of which 1,2,3,4,6 and 8 are single digit factors. Case (i) 8 as one of the digit in the 3-digit number, ‘The product of other two digits is 3. .. The other two digits are 1 and 3. The number are 813 381 138 831 183 318 ‘There are 6 such numbers. 14 Mathematics Talent Tests - Sub-Junior Level Case (ii) 6 as one of the digit in the 3 digit numbers. The product of the other 2 digits is 4. 1. The other two digits are either 2 and 2 (or) 1 and 4. 226 with 2 and 2, the numbers are 262 622 There are 3 such numbers. with 1 and 4, the numbers are 614 164 146 641 461 416 There are 6 such numbers. Case (iii) 4 as one of the digit in the 3 digit numbers. The product of the other 2 digits is 6. “+, The other 2 digits are either 1 and 6 (or) 2 and 3. The combination of 4 with 1 and 6 is already discussed. With 2 and 3, the numbers are 423 243 234 432 342 324 There are 6 such numbers. Case (iv) 2 as one of the digit. ‘The product of other 2 digits is 12. ‘Therefore the two digits are either 2 and 6 (or) 3 and 4, ‘The combinations of 2,2,6 and 2,3,4 are already discussed. Case (v) 1 as one of the digit The product of other 2 digits is 24. Therefore the two digits are 3 and 8. ‘The combination of 1,38 is already discussed ©. The number of such possible 3-digit numbers is, 6+34+6+6=21. . Answer is (C) 31. A four digit number of the form abaa (a’s and b’s are the digits of the four digit number) is divisible by 33, The number of such four digit numbers is (A) 36 (B) 6 (3 (D)1 Screening Test 2004 3 S Sample Qyestions and Solutions 15 Solution: As abaa is divisible by 33, it is divisible by 11 as well as 3. Hence (i) (@+a) ~(b-+a) or ab is divisible by 11 (ii) 30+ is divisible by 3. As a and b are single digited integers (i) implies a - 6 = 0 or a =b and therefore (ii) gives 4a is a multiple of 3. Hence there are three numbers 3333, 666 and 9999 satisfying the conditions of the problem. ©. Answer is (C) The number of three-digit numbers each of which leaves a reminder 6 on dividing by 2002 is A)2 B)4 o)7 D) 10 Solution: Suppose pi,p2--+ are three digit numbers satisfying the condition, having quotients 1,q2,--- respectively when dividing 2002, then (pin + 6) = (pag +6) = ++» = 2002 Pid = poge = 1996 = 2x 998 = 4x 499 Since 499 is a prime number we see that 998 and 499 are the only three digit numbers satisfying the condition. . Answer is (A) 33. Three boys P,Q, R agree to divide a bag of marbles as follows: P takes one more than half of the marbles; Q takes a third of the remaining marbles; R takes the marbles left out now in the bag. ‘The original number of marbles found at the beginning in the bag must be A) a multiple of 6 B) one more than a multiple of 6 16 Mathematics Talent Tests - Sub-Junior Level C) two more than a multiple of 6 D) three more than a multiple of 6 Solution: Let the total number of marbles be 2. P takes 4n+1, ©. x should be a multiple of 2. @ takes } [2 - (Je +1)] =} (}e-3) +, 42-1 should be a multiple of 3. ie, $2—1= 3k (k is a natural number) or = 6k +2. ©. Answer is (C). PART-II 1. A book with 480 pages is to have its pages numbered in the usual fashion. How many digits will this need? From 1 to 9 there are 9x1 = 9 digits From 10 to 99 there are 90x 2 = 180 digits From 100 to 480 there are 379 x3. = 1137 digits Total fto0 ee Therefore, it will require 1326 digits. 2. A book has pages numbered 1 to 192 (totally 96 sheets). Some 25 sheets are pulled out of it at random; show that the sum of these 50 numbers cannot be equal to 2002. Each one of the 25 sheets will have an odd number and an even number, back and back. .'. Total numbers on each sheet is odd. *. Total numbers on 25 sheets is also odd (because odd x odd = odd) .'. the total cannot be 2002, which is even. Sample Questions and Solutions 17 3. Find the number of two digit numbers whose sum of the digits is a single digit number. Final Test 2002 Solution: Possible two-digit numbers Reqd. No. 1 | 10 and 1; but 1 is not a two-digit number 2 | 20,11,02; but 02 is inadmissible a 3 _ | 30,21,12,03; but 03 is inadmissible 3 4 | 40,31,22,13,04; out of which 04 is inadmissible 4 5 | 50,41,32,23,14,05; out of which 05 is inadmissible | 5 6 | 60,51,42,33,24,15 (06 is inadmissible) 6 7 | 70,61,52,43,34,25,16 (07 inadmissible) 7 8 | 80,71,62,53,44,35,26,17 (08 inadmissible) 8 9 | 90,81,72,63,54,45,36,27,18 (09 inadmissible) 9 -. The number of two-digit numbers having the ‘digital sum’ a single digit =1+2+3+---+9=45. 4, Find the sum of all three digit numbers that can be written using the digits 1,2,3,4 (repetitions allowed). Final Test 2004 Solution: We may have any one of the four digits 1,2,3,4 in the hundreds place, tens place or units place. Therefore we have 4x 4x4 = 64 three digit numbers in all. If we consider the units place of these 64 numbers, we see that each of the four digit appears 16 times. The same conclusion is true about tens place and hundreds place. So the sum of the 64 three digit numbers that we can form from the given four digits is S = 16(1+24+3+4) x 100+16(1+2+3-+4) x 10+16(1+243+4) x1 ~*, 5 = 16000 + 1600 + 160 = 17760 18 Mathematics Talent Tests - Sub-Junior Level 5, The natural numbers from 1 to 2100 are entered sequentially in 11 columns, with the first 3 rows as shown. Final Test 2002 C1 | C2 | C3} C4} C5 | C6 | C7 | C8 | C9 | Cio | Cr Rowl{1]2]/3]4]5]6/7]s8}]9] 0] nu Row 2] 12 | 13 | 14 | 15 | 16 | 17 | 18 | 19 | 20 | 21 | 22 Row 3 24 | 25 | 26-| 27 | 28 | 29 | 30 | 31 | 32 | 33 If the number 2002 occurs in column p and row q, find the value of p+q. Solution: Note that 2002 = 11 x 182. There are 11 natural numbers in each and the last entry in each row is a multiple of 11. 2002 will be in “column 11” and “row 182”. p=11 and g= 182 and so p+q=193. 6. In the given addition, each letter stands for a natural number. (Identical letters denote the same number). Find the number for each letter. ONE +FOUR FIVE Final Test 2002 Solution: There are many answers. Two of them are: 3.42 462 +1350 and + 8450 1692 8912 Sample Questions and Solutions 19 7. Observe the following addition problem: ABC > > ww aq iy 4 Q Q Q Each letter stands for a particular number. Find the numbers and rewrite the problem using numbers. Step 1: C+C+C=C. Therefore, C has to be 5 only. 1 ++ ha wy & ane a a a Step 2: Since 3C = 151 is carried over to tens place. A 85 +A 85 +0 Ay 8S. 5.5 5 Therefore, B+B+B+1 should end in 5.-This means B = 8. Step 3: Now 2 is carried over to hundreds place Therefore, 34-+2=5 This means A= 1. 185 ‘Therefore, the problem is * 1 & 5 #185 558 8. Find the sum of all the digits of the result of the subtraction 10° — 99, Final Test 2002 20 Mathematics Talent Tests ~ Sub-Junior Level Solution: 1000000 «+ 0.0000 (1 followed by 99 zeros) : —99 999999 99901 The result is a 99 digit number having 97 nines followed by a zero and al. .. the sum of the digits = (97 x 9) +0+1= 874. 9. This year my age is a multiple of 7. Next year it will be a multiple of 5. I am above 20 years old but less than 80. What is my age? Now my age lies between 20 and 80. It is also a multiple of 7, ©. It should be among the list. 21, 28, 35, 42, 49, 56, 63, 70, 77 q) Next year it should be a multiple of 5. Among the numbers in the list (1), the only number when increased by 1 (to get next year’s age) that becomes a multiple of 5 is 49. (Others fail to satisfy this: for example 21+1,28-+1,++- are not multiples of 5. Only 49 +1 gives a multiple of 5) . Required age is 49. 10. Let us assign natural number values to English alphabets as follows: A=1,B=2,0 =3,D=4,E=5,+,2=2. Final Test 2003 Solution: Let us define the ‘value’ of a word as the ‘product? of the numbers represented by the letters in that word. (For 11. Sample Questions and Solutions 21 example, the ‘value’ of CAT is 3x 1x 20 = 60). Find a word whose ‘value’ is 455. 455= 5x7x13 444 EGM The word required must be made of E,G and M. We can also have as many A’s if required. GEM is one such required word. MEGA is another word. (Multiple answers are possible) What is the smallest number n greater than 1 such that vI+2F3+--- +7 is a positive integer? Solution: Given: /™2+ Final Test 2003 x a positive integer. Values of n'| Values of n(n + 1)/2 2 2277) 5, not a square 3 = 6, not a square 4 5 6 7 8 seen = 36, a square. «', The required n = 22 Mathematics Talent Tests - Sub-Junior Level 12. A six-digit number of the form ABCABC is always divisible by 7 or 11 or 13. Explain why. The number of the form ABCABC is divisible by 7 or 11 or 13 i.e it is divisible by 7 x 11 x 13 = 1001. If we have any three digit number of the form ABC’ and multiply it by 1001, we will get ABCABC ie ABCABC is divisible by 7, 11, 13. (Example: 987987 = 987 x 1001 = 987 x 7 x 11 x 13] 13. Given 72¢ = A679B where A,B are unknown whole-number digits, find X. A679B is 72 times x. *, A679Bis divisible by 8. i.e., A6000 + 79B is divisible by 8. ie 79B is divisible by.8. .. B is 2 (by trial). [Note that B cannot be any other one-digit number] Again, since A679B is 72 timesz, A679B is divisible by 9. Therefore, A +6+7+9+ B is divisible by 9 ie A+ 22+ B is divisible by 9 ie A+24 is divisible by 9 ( because B is 2) ie A=3. , 12x = 36792 _ 36792 = 2078 «su ere 14. Using only the digits 0,1,2,3,5 (with no repetitions), three- digit numbers are formed. How many of them are multiples of 6? Final Test 2003 Solution: Hundreds | Tens | Unit z y * z should be 2 or 0 (because if the number is a multiple of 6, it should end with 0 or 2 among the given numbers). Sample Questions and Solutions 23 2+y+2z should be a multiple of 3. Ifz=0, thnz=ly=2 Ifz=2, thne=ly=0 or z=2,y=1 or c=ly=3 or c=1y=5 or t=3,y=1 or r=5,y=1 Thus there are 7 possibilities. (Note: The numbers required would be 120, 210, 150, 510, 102, 132, 312). 15. Find the quotient of the least common multiple of the first 40 natural numbers divided by the least common multiple of the first 30 natural numbers. Final Test 2003 Solution: tional fact LCM of LOM of ee eae the first (J the first pate taes 40 natural [ ~ } 30 natural Fae of primes not numbers numbers already present first 30 natural fifth 2 which is LCM of the 31,37 and the = x soumbers a factor of 32 Lc.m of the first 40 natural numbers Tem of the first 50 natural numbers ~ °!% 37x 2= 2294 16. Find an integer n that leaves remainders 2, 3, 4 when divided by 3, 4, and 5 respectively. Final Test 2002 Solution: n+3 gives remainder 2 => (n+ 1) +3 will give remainder 0. = (n+1) is divisible by 3. Similarly, (n + 1) is divisible by 4 and 5 also. (n+1) is divisible by 3,4, & 5 => (n+1) is divisible by the Leam of 3,4, & 5. = (n+1) is divisible by 60. , n-+1=60 and hence n = 59. 24 Mathematics Talent Tests - Sub-Junior Level Note that there can be many answers for n. However 59 is the least number satisfying the given conditions. 17. The greatest common divisor of a and 72 is (a,72) = 24 and the least common multiple of 6 and 24 is [6,24] = 72. Find the g.c.d (a,b) and the l.c.m (a,6] given that a is the smallest three digit number having this property; and 6 is the biggest integer having this property. Final Test 2004 Solution: The smallest possible value for a is the smallest 3-digit number which is a multiple of 24; hence the smallest possible value for a is 120. The biggest integer b with the property [b,24] = 72 is clearly 72. So for the desired numbers a,b we have g.c.d (a, 5) = g.c.d(120,72) = 24 and 1.c.m{a, 6] = Lc.m[120, 72] = 360. 18. The sum and least common multiple of two positive integers z,y are given as 2+y =40 and lc.m|[ 2,y] = 48. Find the numbers x and y. Final Test 2004 Solution: Let the g.c.d(z,y) be d. Then z = do and y = db for some integers a,b. We have z+ y = d(a + 6) and we observe that a and 6 have no common factors other than 1. Now, Le.m[z,y] = 1.c.m[da, db] = dab = 48. Again, _g.c.d (d(a +b), dab) = d x g.c.d((a + 6), ab) =d since g.c.d((a+6),ab) = 1. This means that g.c.d (40,48) =8=d. Therefore, we get a+b=5, ab=6 which on solving give a = 3, b=2 or a=2, b=3. On substituting these values for a,b we get r= 24, y=16 or c= 16, y= 24. Sample Questions and Solutions 25 19. What is the greatest positive integer n which makes n3 +100 divisible by n+ 10 ? Final Test 2004 Solution: We have n® +100 = n? + 105 — 900 = (n + 10)(n? — 10n + 100) — 900. Therefore n+10 divides n3+100 ifand only if n+10 divides 900. The largest n such that n+ 10 divides 900 is 890. So, the answer is 890. 20. The ten digit number 3455B1063C is a multiple of 792. Find A,B,C. 3A55B1063C is multiple of 792. .’. it is multiple of 9 or 11 or 8 since 792 = 9 x 11 x 8 since it is divisible by 8. The last three digits are divisible by 8. ie., 63C is divisible by 8. , C must be 2. (1) Since it is divisible by 9, 34+A454+5+B+140+6+3+C isa multiple of 9. ie. 23+A+B+C isa multiple of 9. .c. 25-+A+B isa multiple of 9 (- C=2) (2) Since it is divisible by 11 3A55B10632 3+5+B+0+3 and A+5+1+6+2 differ by 0 or 11. ie, (11+B) and (14+ A) differ by 0 or 11. (14+ A) - (11+ B) =0 or (14+A) —(11-B) = 11,22,--- A-B=-3 or A-B=8,9,-. (Of these A—B = 19 etc are ruled out;) Only A~B=-3 or A~B=8 are possible (3) If A-B Then A = B-3, and by (2) 25+ B-34+B isa multiple of 9. ., 2B +22 is a multiple of 9. . Single digit B must be 7 (then 2B +22 = 36) and also A=B-3=7-3=4 v. In this case A=4,B =7,C =2. 26 Mathematics Talent Tests - Sub-Junior Level If A-B=8. Then A= B+8, and by (2) “.25+B+8+B isa multiple of 9. ie, 2B +33 is a multiple of 9. <. Single digit B must be 6 (then 2B + 33 = 45) and also A=B+8=64+8= 14 whichis not possible since A must be a single digit. ©. The only possibility is that A =4,B=7,0 =2. [In which case the number would be 3455710632 and it is easily verified that this is divisible by 9,11 and 8 and hence by 9x 11 x 8 =792] 21. Rekha was asked to add 14 to a certain number and then divide the result by 4. Instead she first added 4 and then divided by 14. Her result was 5. Had she followed the instructions correctly, by how much would her result have differed from the incorrect result? Final Test 2002 Solution: Let the number be z. Rekha’s Solution: As i = 5, « should be 66. Correct Solution: S414 = 20 ., Rekha’s answer was 15 less than correct answer. CHAPTER - II PATTERN DETECTION PART-I 1, A student starts at the year 2001 and counts backwards, 8 at a time, giving the sequence of years: 2001, 1993, 1985, --- A year which she will count is A) 1841 B) 1901 C) 1923 D) 1903 2001 when divided by 8 gives remainder 1 1993 when divided by 8 gives remainder 1 1985 when divided by 8 gives remainder 1 ‘Therefore, among the given answers, the one that gives a remainder 1 (when divided by 8) should be correct. Therefore, 1841 is the correct answer. <. Answer is (A). 2. Match-sticks are used to form the following pattern made of oO OOO In this pattern, how many match-sticks are needed to form squares: ten squares in a row? A) 36 B) 40 ©) 31 D) 30 Screening Test 2003 27 28 Mathematics Talent Tests ~ Sub-Junior Level Solution: Let’s tabulate as follows. number of number of number of Figure no. match sticks — match sticks match sticks at the top at the bottom that are vertically placed 1 1 1 2 2 2 2 3 3 3 3 4 10 10 10 11 By pattern detection, the 10‘ figure will have 10+ 10 + 11 =31. match sticks. Alternate Method: 1st figure has (1+3) match sticks 2nd figure has (1+3+3) match sticks 8rd figure has (1+3+3+3) match sticks ja.-+ bal 14+3 14+34+3 and so on. 10 figure will have 1+34+3 ten threes plus one 3=31 match sticks. Alternate Method: 10 figure have 10 squares that corresponds to 10 x 4 = 40 match sticks of which there will be 9 common match sticks. *. 10 figure will have exactly 40 — 9 = 31 match sticks Answer is (C) Sample Questions and Solutions 29 3. The sum 1-2+3-4-4-.--+47—48 +49 is A)O B)24 ©) 25D) #580 Screening Test 2003 Solution: 1-2+3-4+5-6+4---+47=48 + 49 (24 pairs of — 1) +49 = -24+49 = % Aliter: 1-2+3-44+5-6+7---—46+47—48+49 = 1+(-24+3)+(-44+5)+(-6+7)+°- +(—46 + 47) + (—48 + 49) = 1+ (24 pairs of 1) 14+24=25 ©. Answer is (C) 4, Look at the following pattern: The value of 1+3+5+---+2003 is the number of the dots appearing in the A) 1002"¢ diagram and is equal to 1002? B) 1001%t diagram and is equal to 1001? C) 2003" diagram and is equal to 2003? D) 2002"4 diagram and is equal to 2002? atts 2 Ist diagram: 12 = 1 = (+4) 2 Qnd diagram: 22 = 143 = (4) 2 3rd diagram: 3° = 14345 = (4) 30 Mathematics Talent Tests - Sub-Junior Level 2 2003 = (4) = 10027 which will be in 1002" diagram. ©. Answer is (A). SLt3454- 5. In the sequence 1,22,333,4444,---, the n‘* term has n number of n’s. Then the sum of the number of digits in the 100" term is (A) 100 (B) 300 =~ (C) 10000 (D) 100" Solution: 100 term constitutes blocks of hundred 100s. ~. To write the 100" term we use 3 digits for each hundred. “, we use (100 x 3 =)300 digits in total. Sum of these 300 digits = sum of the numbers of 1" used =100x1=100 ©. Answer is (A). PART-II 1. Natural numbers are written in a sequence as follows: 1234567891011 12 13 1415 16 --- What is the 2003rd digit in this sequence? Final Test 2003 Sample Questions and Solutions Solution: ‘The digits 1,2,---,9 take the first ‘The digits 10,11,12 --- occupy The digits 100,101,-- occupy The digits 200,201,- occupy ‘The digits 300,301, -- occupy ‘The digits 400,401,--- occupy ‘The digits 500,501,--- occupy ‘The digits 600,601,- occupy 9 2x 90=180 3x 100 = 300 3x 100 = 300 3x 100 = 300 3 x 100 = 300 3x 100 = 300 3x 100 = 300 position position position position position position position position To get 2003rd position we need another 14 positions. We are required to write 700 701 702 703 70. The 2003rd digit in the sequence is 0. 31 189 489 789 1089 1389 1689 1989 CHAPTER ~— III RATIO AND PERCENTAGE PART-I “1. A and B are respectively 20% and 10% lower than C, What percent of B is A? A) 66.66 B) 88.88 ©) 50 D) 45 Screening Test 2002 Solution: 80 90 A= {0 = 80; B= 750 =.9C A_8C_8 8 Bx GT 5g * 100% = 88.88% ©. Answer is (B). 2. The largest value among 4,80%,0.801 and (.9)? is A) ¢ B) 80% C) 0.801 D) (.9)? Screening Test 2003 Solution: 4 = = 0.800 5 80 = — =0.800 80% i00 72 0.801 = 0.801 (0.9)? = 0.810 (0.9)? is the largest among the 4. <. Answer is (D) 32 Sample Questions and Solutions 33 ’ 3. If 10% of m is the same as 20% of n, then m:n is A) 5:1 B) 2:1 ©) 12 D) 10:1 Screening Test 2002 Solution: 10S wih ™* 700 = "” Too m _ 20, 100 _2 Femi 4100-7 Oh aL min = 2:1 ©. Answer is (B). 4. If a+2a+3a+---+ 10000 = 2b + 4b + 6b +.... + 20006 = 3c+6c+9c+-+++3000c then a:b: is as (A) 1:2:3 (B) 3:2:1 (©) 2:3:6 = (D) 6:3:2 Screening Test 2004 Solution: Cancelling off the common factor 1+2+3+---+ 1000 from the three equal sums given, we get a_b_e a=2=3eor e= 5 z a:b:c=6:3:2, — , Answer is (D) 5. Scientist A has invented a device for cars to save petrol by 20%; Scientist B’s invention saves 30%; invention of scientist C saves 40%. The three inventions are independent in effect. If all the three devices are used, how much petrol can one save? A) 90% B) 140% = C) 66.4% +~—-D) 33.6% Petrol saved = 100% — (100% — 20%) (100% — 30%)(100% — 40%) 100% — (80% x 70% x 60%) 66.4% W 2. Answer is (C). 34 Mathematics Talent Tests - Sub-Junior Level 6. Given X is 50% larger than Z and Y is 25% larger than Z, Then X is what percent larger than Y? A) 20% B) 25% ©) 75% D) 50% 180 100 125 Typ _ (Therefore ¥ is 25% larger than Z) (150) 150 _ 6 (353) 100 «5 = 1.20 _ 120 ~ 100 (Therefore X is 50% larger than Z) W " =12 Bee ~.X is 20 % larger than X. ©, Answer is (A) . 7. A chocolate drink is 8% pure chocolate, by volume. If 5 litres of pure milk are added to 25 litres of this drink, the percent of chocolate in the new drink is nearly A) 10 B) 7 ©) 15 D) 13 ‘Screening Test 2002 Solution: The quantity of pure chocolate in the final mixture is 8% of 25 litres, ie. 7 x8 =2 litres. «. In the final mixture, (after adding 5 litres of milk), pure chocolate will be 2 litres out of (25 +5) = 30 litres. ~. Required percentage = 3 x 100 =7 (nearly). . Answer is (B). 8, If the price of an article is increased by 15% and the total number of articles soled is decreased by 10% , the profit on income is A) 5% B) 1.5% C) 3.5% D) 2.5% Sample Questions and Solutions 35 Assume: Original | New No of Articles | 100 90 Cost of Articles | 100 115. Original income = Rs.100 x 100 = Rs.10000 Revised income = Rs.90 x 115 = Rs.10350 Therefore, profit = 32535 x 100% = 3.5% (Note : Formula : Effect on income = (pm — 1%) - PAX 15 x 10 = (15-10) - 7% = 3.5% Can you derive the formula?] . Answer is (C). 9. In an examination 60% passed in English, 52% passed in Maths while 32% failed in both. If 220 candidates passed in both, then the total number of candidates was A) 200 ~—-B) 300 C) 400 =D) «500 Screening Test 2002 Solution: Let « number pass both in English and Maths. From the adjacent diagram, Percentage of failure in either English or Maths or both 60-r+2+52-2 = 12-2 68 = 12-2 org = 44 This 44% is 220. .*. Total 100% = a x 100 = 500. 36 «, Answer is (D). Aliter: % failed in English but not in Mathematics % failed in Mathematics but not in English % failed in both % passd in both Thus 44% of the. candidates Total number of candidates = 720%10 — 500. Mathematics Talent Tests - Sub-Junior Level 40 -32=8 48 —32=16 32 100 — 8 - 16-32 = 44 is 220. s. Answer is (D). 10. If x is § of 490, then $ of a is A) 350 B) 250 ©) 70 D) 420 Screening Test 2003 Solution: Given 5 2 = 2x 490 7 5 5S. Bxe = px 7% 490 _ 5x5x(7x7)x10 fi (7x7) = 250 Answer is (B) 11. A brick weighs 3"* the weight of itself plus 3kg. The weight. of the brick (in kg) is A)2 B) 2.5 oo D) 2.75 Screening Test 2003 Solution: Let the weight of the brick be x kgs. Given 3 xatd + jxe 1. Answer is (C) x=3 ale 8 (or) Sample Questions and. Solutions 37 12. Four points P,Q, R,S are on a line segment, as shown P Q R s If PQ:QR=1:2, QR: RS=8:5, then PQ:QS is A) 3:13 B) 1:7 ©) 1:13 D) 4:13 Screening Test 2003 a Solution: Given PQ:QR=1:2 and QR: RS=8:5 (or) PQ:QR=4:8 PQ:QR:RS = 4:8:5 (or) PQ:(QR+RS) 4:(8+5) => PQ:QS = 4:13 .. Answer is (D) 13. $1, S2, $3 are three sums of money such that Sp is the simple interest on $; and S3 is the simple interest on S> for the same rate and same period. Then A) 5S? = S253 B) S} = SiS, C) S23 = 55, D) 915253 =1 Screening Test 2002 Solution: Let the period be n (years) and the annual rate be r% Sp is the simple interest on Sy. Syxnxr — = So (1) Sy is the simple interest on Sp. _ Sxnxr 1 at es (2) S2 ee te Ss Dividing (1) by (2), 5 = Ho SP = S55, ©. Answer is (C), 38 Mathematics Talent Tests - Sub-Junior Level PART-II 1. From a group of boys and girls, 15 girls leave first. Then the ratio of the number of girls to the number of boys becomes 1: 2, After this, 40 boys leave the group. Now the boys and girls are equal in number. How many girls were there in the beginning? Girls:Boys In the beginning When 15 girls leave When 40 boys leave | B-40 (G-15):B (G-15):(B-40) Given G-15 = B-40 (1) G-15:B=1:2o0r2x(@-15) = 1xB ie, 2G-30 = B (2) Substituting in (1) the value of B in (2), G-15 = 2G-30-40 =>G = 55. ‘There were 55 girls in the beginning. 2. Four persons A, B,C,D went to a hotel to take tiffin. The total bill was Rs.60. ‘A agreed to pay half the sum of the amounts paid by the other three; B agreed to pay one-third of the sum of the amounts paid by the other three; C agreed to pay one-fourth of the sum of the amounts paid by the other three; Sample Questions and Solutions 39 How many did D pay? Amount paid by A: Amount paid by B,C,D::1:2 ‘Thus A paid } of the sum i.e Rs.20 Similarly B paid } of the sum i.e Rs.15 C paid } of the sum ie Rs.12 Therefore, D paid Rs. 60 - (20+15+12) = Rs.13. 3. The sizes of copier paper have the property that a sheet of paper cut in half gives two smaller sheets of the same shape as the original sheet. Find the ratios of the sides of the sheets. nie np Final Test 2003, Solution: Let the sides be in the ratio: x:1. When folded we get the ratio 1: F. or ie, r=v2 .". The sheets’ sides are in the ratios V2:1. 4. Place one non-zero digit in each box given below in such a way that the resulting equation is valid: OO % of GOO = 400 Final Test 2003 Solution: Given a % of b =400 where a is a two digit number and 6 ‘is a three digit number. a a% of b= 75 x b = 400 => ab = 40000 = 2° x 5! 40 Mathematics Talent Tests — Sub-Junior Level Since a and 6 consist of non-zero digits only, 10 cannot be a factor of a or 6 and neither can have both 2 and 5 as factors. 28 — 64 and b= 5! = 625. [Clearly 64% of 325 is 400). When the price of a gas cylinder is increased by 20% , by what percent should a householder reduce his consumption such that there is no increase in his expenditure? Revised price = 32° of the original price. ©. Present consumption = 129 of original consumption. Reduction in consumption = (1— 398) of the consumption = 4 of the original consumption = 163% ‘The ‘length of the base of a rectangle is increased by 10% but the area remains unchanged. By what percent was the breadth reduced? “Original Ctr Length Breadth Length Breadth L B Lik? Area is not changed Therefore L x B = (L+ 4) (B- 8) [where x is the % reduced in breadth] bf Br ~ Box (a2) 100 u z LxB = i5xbx(1-s5) xB rw By ios = yo * To We ye Tor 100 Therefore x = 9% Sample Questions and Solutions 41 7. There were four tests in Mathematics. A student scored 84, 78 and 76 in the first three tests, each out of 100. How many marks should she score out of 200 in the fourth test so that the average on all the four tests would be 90% 7 She wants to score 90% (out of 500 marks). Therefore, she has to score (4 x 500 =)450 marks. Total out of 3 tests earlier = 84 + 78 + 76 = 238. ‘Therefore, she needs (450 - 238=) 212 more. This is not possible in the last test. It is not possible for her to get the required score. CHAPTER — IV CLOCKS, TIME AND DISTANCE PART-I 1. The time on an electronic digital watch is 11.11. How many minutes before this would the watch have shown a time with all digits identical? A) 72 B) 144 C) 216 D) 316 Screening Test 2002 Solution: The required digits on the clock before 11.11 would be 5.55. This gives a time difference of 316 minutes. (Note that times like 6.66, 7.77 are impossible). , Answer is (D). 2. A watch is set right at 3 pm. It loses 20 minutes in 24 hours. The true time when the watch shows 2 pm on the fourth day is A)3pm. B) 4.19pm. C) 3.19pm. D) 4 pm. For 3 full days, it loses 3 x 20 min -1 hour. 4th day, for 23 hours it loses 20 min. —(33 x 20) min = 20 min — 50 sec. nearly = 19 min10 sec. .. True time will be 2.00 pm + ihr.19 min. ie. 3hrs.19 min. <, Answer is (C). 42 Sample Questions and Solutions 43 3. Ina kilometre race, A beats B by 1 minute and beats C’ by 375 metres. If B beats C by 30 seconds, the time taken by C to run 1km is A) 150 sec B) 210 sec. © C) 240 sec. ~—-‘D) (200 sec Screening Test 2003 Solution: Let's analyse this problem by pictorially. @©®— i nin | 30S¢e \ | (375 meters Assuming that B,C are travelling at the same speed, the distance between B&C’ would be same through out. (After A finishes the race). From the picture, Time taken Distance travelled (in seconds) (or) can be travelled by C (in meters) 90 375 +90 375 180 750 3> 60 +3=> 250 Adding 240 1000 m . © could run 1 km in 240 second. :. Answer is (C) 4, Sound travels at 330 m/s. How many kilometres away is a thunder cloud when its sound follows the flash after 10 seconds? A)33 B) 33 ©) 0.33 D) 3.33 44 Mathematics Talent Tests - Sub-Junior Level Distance = speed x time = 330 x 10 metres — 3300, * {000 = 33km ©. Answer is (A) 5. Which one of the following distance time level graphs is not possible? Graph (D) is not possible, because it shows different positions of the same person at the same time! ©. Answer is (D). PART-II 1. Ina twelve-hour period of time, find the duration (in minutes) when the number indicating the hour is greater than the number indicating the minute. Time (hours) | Duration, when hour > min From 12.00 to 12.11 12 Min 11.00 to 11.01 11 Min 10.00 to 10.09 10 Min Sample Questions and Solutions 45 Therefore, required number (in min.) =124+114+10++--+1= 78. 2. A clock gains seven and a half minutes every hour. It was set correctly at 12 noon on Sunday. When and on what day at the earliest would it show correct time? Final Test 2003 Solution: The faulty clock gains 7} min per hour. i.e., 3 hrs per day. The position is as follows: Day Correct Time ] Time shown by the clock Sunday _ | 12.00 12.00 Monday _| 12.00 3.00 Tuesday _| 12.00 600 Wednesday | 12.00 9.00 Thursday _| 12.00 12.00 At the earliest the clock will show the required time on ‘Thursday. 3. We know that a clock’s hands coincide when the time is twelve o’clock. What is the next time that the clock’s hands coincide? At 12 o’clock, the hands coincide. Then the hands start moving. Since the minute hand moves faster, the hands will not coincide before it makes a complete cycle. ‘Therefore, the minute hand continues to move, chasing the hour hand, Before the minute hand makes one more round, the hands will meet. At the time of their meet, let the minute hand had made one complete round and a fraction of a round, say k. 46 Mathematics Talent Tests ~ Sub-Junior Level Meanwhile the hour hand would have travelled the fraction k of one full round only. We know that the minute hand moves 12 times faster, when compared to hour hand. Therefore, 12k = 1+ ie k= x of a full round, It takes one hour for the minute hand to complete one round. ‘Therefore, after 1 hour and $? minutes, the two hands will coincide again. 4, A military officer sends a secret from a military base A to base B, where AB = 29km., through a messenger travelling at 40km/hr on a vehicle, After 15 minutes, the officer plans to stop the messenger before reaching B, by sending another messenger at 60km/hr. Will the plan work? Let P be the Ist messenger and Q is the 2nd messenger? 4 : 2 (enRESeEESRES tn Sess Ssassasesaenet Beginning a 7 29 kn After 15 minutes —— Qitere 10k In 15 minutes messenger P would have covered a distance 1 x 40km = 10km. . By the time Q starts from A, the messenger P would have gone 10km towards B. Now Q starts. The difference, at Q’s start, between P and Q is the distance 10km. Difference in their speeds. = (60km — 40km)/hr = 20km/hr. If Q were to meet P, he has to make up this difference in distance. Sample Questions and Solutions 47 ‘Time taken by Q to meet 2 Distance between them Difference in speeds = 2h 20 = 30 minutes (after Q’s start) Note that by the time A meets P, the messenger P has actually travelled for (15+30 =)45 minutes. In 45 minutes, P would have crossed (the distance of 29 km) place B So Q cannot catch P in time. The plan will not work. CHAPTER —- V INDICES PART-I 1, What is half of 4°? A) 20 B) 4” ©) 2” D) 27 40 yi? «. Answer is (C). 2. The value of } of 15°? is A) 577 -B) 159 GC) «5 x 15% =D) 5 x 39 Screening Test 2002 Solution: 2 x 15% = i x 15 x 15% = 5 x 1576 «. Answer is (C). 3. Which one of the following number is perfect square? A) 455465 ~—-B) 4°5465 = C) 445565 =D) 445665 Screening Test 2003 Solution: Consider 455468 (2?)8 x 54 x 68 2! x 54 x 68 = (25 x 5? x 6%)? is a perfect square. ©. Answer is (A) 48 Sample Questions and Solutions 49 4, If a?bc3 =5% and ab? = 5°, then abe equals A)5 -B) 5 Qo) 5 D) 548 ; . Screening Test 2002 Solution: Multiplying together, (a?bc3)(ab®) = 5° x 5° = a®b?c3 = 5° => abe =5°. o. Answer is (C). 5. If 44 +444 4 444 4 444 — 4? then a is A) 45 B) 44 C) 176 D) 1 Screening Test 2003 Solution: Given 44444444444 = 4? = 4x4 = 4 om we (or) gs = gt > z= 45 ~. Answer is (A) +e sage +6 +65 +654 65465 6. If —a54 95 ——35-5 3535 = 8" then, n is A)l B)2 c)3 D)4 48 68 pre =e Therefore, 48 = gn (27)8 = (28) 122 = 9* n= 4 <. Answer is (D). 50 Mathematics Talent Tests - Sub-Junior Level 7. If 2% x 3? = 576 then ¢ is A)2 B)3 ©) 4 D) 4} 2 x 3° = 576 =2 x37... a= and b=2. 1 a$b=642=3. ©. Answer is (B). 8. Given n is a natural number. If prt n?tn? tn? +--- +n? = 64, then 1 times Binitndi+nd+--- tn =? n? times A) 512 B) 256 ©) 1024 D) 2048 ne tn? tn? +---(n times) =n xn? =n3 v.n8 = 64=4x 4x4 which gives n= 4. nr +n3+n3+-+-(n? times) =n? xn? = nd. w. Required value =n5 = 45 =4x4x4x4x4= 1024 . Answer is‘(C) 9, The total number of powers of prime factors in the expression 67 x 35!” x 33°7 is A) 24 B) 44 C) 121 D) 102 6" x 35'7 x 337 = (2x 3)" x (5 x 7)!7 x (3 x 11)” 2 x 37 x BT x TIT 5 927 x 1177 = 2% x 34 x 517 x 7H? x 117 No. of prime factors = 7-+34+17-+17-+27 = 102 Answer is (D). 4714571, thon x is A)? B) 3 ©) 2% D) # Screening Test 2003 Sample Questions and Solutions 51 Solution: Given 27) = 4457 ae zs 475 Se bea Pr) > r= a 7a ~. Answer is (A) PART-II 1. Find the integers x,y if 29° — 377 = 55. (28)? — (3)? = 55 (2? +39) (2 — 3%) = 11 x 5. 2 4+3¥=11 24 BV=5. 1.27 -WHS or 27 -3¥=11. ADD: 2x 2? = 16. oF =F = Bi o=3. If 7 = 3,29 +39 =113Y = 11-8. 3y=3 > 3 = 3! and ere ©, Solution is 2 = 3,y = 2, What is the last digit when 370 is evaluated and written as in the regular number form? 3! = 3;37 3 = 27; 34 = 81; 3° = 243; 36 = 729; The only possible ending digits (of powers of 3) are 3,9,7 and 1. etc. 52 Mathematics Talent Tests - Sub-Junior Level After every 4‘* power, the pattern repeats. 3* ends in 1 38 ends in 1, 32 ends in 1 and so on. 37002 — 32000 32 — (34)500 x 9 — ( Number ending in 1) x 9 Therefore, 370°? ends in 9. 3. What is the number in the unit place-of (2003)20027 2003? has 9 at its unit place. 2003! has 1 at its unit place. Therefore, 2003702 = 200379 x 2003? = -20034)500 my 2 (2003") , 2003 ; has 1 in the units place has 9 in the units place Therefore 20037 has (1 x 9= 9) in the units place. CHAPTER - VI ALGEBRA PART-I 1. Ina contest there were 10 problems. Each correct answer is eligible for 5 points while each incorrect answer is penalised with a deduction of 2 points. Venkat answered all the questions but got only 29 points. The number of correct answers of venkat was A)5 B)2 C)7 D)4 Let the number of correct answers be <. So, no. of incorrect answers = 10-2. 5 marks for each ¢ and —2 marks for each of (10—z) gives 5(z) + (-2)(10—2) = 29, = 5-204 22 = 29 > 72 = 49 > x = 7 (Now verify your answer!) ©. Answer is (C). 2. If a=2,b=3a+4 and c= 3b, then b—c is A) 30 B) 20 ©) -20 D) -22 Screening Test 2002 Solution: = 8(3a+ 4) =3(6 +4) = 30. ss = ; x 30 = 10. Thus, = 10-30=—-20. «. Answer is (C). 3. If 2a-3=5 and 35+1=2. then 3b—2a is A) -3 B) -7 C)7 D) -9 Screening Test 2003 53, 54 Mathematics Talent Tests - Sub-Junior Level Solution: 2a-3=5; 3b+ => 2a =8 and 3b=1. J. 3b-20=1-8= «, Answer is (B) 4. If a—1=b+2=c—3 =d+4 then the largest among a,b, c,d is Aja B)b Oc D)d c-3=a-1. c-a=2andsoc>a c-38=b+2 c-b=5andsoc>b So,c is the greatest. c-3=d+4 c—d=Tandsoc >d . Answer is (C). 5. If ae=a+1 and +a=a—1, then 1¥—*14+24—*243+ — «3+... +1000 * — * 1000 is equal to (A) 1000 © (B) -1000 += (C) 2000 (D) ~2000 Screening Test 2004 =1 we get ava «+1000 terms = 2000. Solution: As at =a+1, +a Thus the given sum is 2+2+ . Answer is (C) 2 6. Watb-+em0, then -to*™ ip ca A)O B)1 C)2 D) 3b Screening Test 2003 Solution: Given a+b+¢=0 => b=-[a+e] Sample Questions and Solutions Squaring on both sides, we get P =a? +c? +2ac 55 P+et+a? 2a? +c? +a) Rca (a? +c? +.ac) ©. Answer is (C) 7. If > 2y and z< $ then A) a>6z B) «<6z ©) y<$ Solution: a> dy 5 > 2 y>3z. From (1) and (2), 2 > 2(3z) or 2 > 6z. ©. Answer is (A). 8. If r=9+4V65 and cy=1 then y+ % is D) z>& @ (2) A) 81 B) 322 ©) 97 D)2 Screening Test 2003, Solution: es y= 1__1_ 9-4v5 ZT 944/56 9-475 9-45 Fy = grag 79 4v5 = 1 2 = (e+2)7-2 = (e@+y)?-2 = 18?-2 = 322, .:. Answer is (B) 56 Mathematics Talent Tests - Sub-Junior Level 9. At the end of the year 2002, Ram was half old as his grandpa. The sum of the years in which they were born is 3854. Age of Ram at the end of year 2003 is A) 50 B) 35 C) 51 D) 36 ‘Screening Test 2003 Solution: Let's analyse the problem by pictorial approach. —a2—+ yrs|—2—$ yrs Grandpa’s Ram's 2002 birth year (y+2) birth year (y) year Assumptions are made according to the data given. Given, yty+2 = 3854 () from picture, y+ z+ = 2002 (2) Adding (1) & (2), we get 3(y+2) = 5856 > yto = 1952 . Ramu was born in the year 1952. His age at year 2003 will be 2003 — 1952 = 51. :. Answer is (C) 10. Given distinct single digits A,B,C,D,E,X,Y suppose we have A+B+C+D+E = XY where the two-digit number XY has value 10X+Y. To make XY a greatest value, the value of Y should be A)4 B)3 c)2 D)1 To make 102-+y greatest possible value, A, B,C, D, B, (The distinct single digit numbers) should be as large as possible. Let us list the various possibilities and see it the possibility is permissible. Sample Questions and Solutions 57 ‘Trial values for Valiie 10X+Y Is it OK? ABCD. E A ,C=7,D=6 E=5 35 Not permitted since . digit 5 repeats A=9, B=8,C=7,D=6 B=4 34 No (4 repeats) E=3 33 No (3 repeats) 32 No (2 repeats) 31 No (1 repeats) =5 34 No (5 repeats) E=4 33 No (3 repeats) A=9, B=8,C=7,D=5 E=3 32 No (3 repeats) A=9, B=8, C=7,D=5 E=2 31 Yes(No repetition) A=9, B=8,C=6,D=5 E=4 32 ‘Yes(No repetition) From the last two we find the last one is correct. <. Answer is (C). 11. Given positive integers a, and distinct positive prime numbers pi,p2. If at —b* = pip2 then a—6 is A)1 B)0 C)aprime number D) a composite number By the given condition (a? — 0)(a? + 8) =p x po (a —6)(a +6)(a? +b) =1x py x po Clearly a > b and (a—b) > (a+b) > (a?'+8?). Thus a—b should be 1. v. Answer is (A). 12, ‘a? and ‘b? are two natural numbers with a+6 = 8. If a>b and a? +6? has minimum value, then @ and 6 are given by (A) 7,1 (B) 6,2 (©) 44 (D) 525 Screening Test 2004 58 Mathematics Talent Tests ~ Sub-Junior Level Solution: a+b? = a? +(8—a)? = 20? — 160464 2(a? — 8a + 16) + 32 = 2(a — 4)? +32 This is minimum when a= 4. Then 6=4. ©. Answer is (C) 13. The product of Hari’s age in years on his last birthday and his age now in complete months is 1800. Hari’s age on his last birth day was (A)9 (B) 10 (oC) 12 (D) 15 Screening Test 2004 Solution: If Hari is x years old on his last birthday and y < 12 be the number of months elapsed after that birthday, we have (12x + y) = 1800. 122? < 1800 or 2? < 150. ‘The largest integral value of « satisfying this inequality is 12. ‘Thus Hari’s oge on his last birthday =12 years. ©. Answer is (C) 14. Ram is 7 years younger than Ravi. In four years time, Ram will be half of Ravi’s age. The sum of their ages now is (A) 13 (B) 15 (C) 17 (D) 19 Screening Test 2004 Solution: If the present age of Ram is years then that of Ravi is 2-+7 years. After 4 years their ages will be = +4 and z+11. As Ram’s age after 4 years is half of Ravi’s age then, we have Ar+4)=24+11. Hence r= 3. So the sum of their present ages is 3+ (3+ 7) = 13. vt, Answer is (A) 15. Sample Questions and Solutions 59 Aliter: The situation is diagrammatically represented below: Ram Ravi —_—__ oO A ?YersB Ravi —_._____. oO A#Years4' 7 Years = B Ram Four years hence Ram's age is represented by OA’ and Ravi's age is represented by OB'. Sine OA’ =half of OB’, OA! = A'B! =Tyears. OA =OA'—AA' = 7-4 =38 years and OB = 3+7 = 10 years. Hence sum of their present ages = 3+ 10 = 13. Let a#b=a-+b-+ab where a,b are rational numbers, neither of them equal to -1. For example 6+2=64+24+(6x2) = 20 1 1 1 i: W2e a2 T+ (2x G) = 25. If ax2=0, then o is A) at B) = CQ) D) 3% Screening Test 2002 Solution: axez=0 > at+tz+ar=0 > 2(l+a)= aor r= l+a ©. Answer is (D). . Instead of multiplying a given number by §, a student divided it by . His answer was 297 more than the correct answer. The given number was A)8 B) 19 ©) 152 D) 64 60 Mathematics Talent Tests - Sub-Junior Level Let the given number be z. 8 8 +3)_(2x+) = 297 rete (2+ 8)-(ex 8) = 2 19 8 )_(2,2) = 297 a {ax#)_{ox2) 19 8 19_ 8) = o97 wro(2-8) =m (361-64) _ og 152 297 axa = 297 ° 1 a = 27x 5 = 152. ©. Answer is (C). PART-II 1. A question paper has n questions;, (n > 20). Out of the first 20, a student answers 15 correctly and out of the remaining he answers one-third correctly. If all questions carry equal marks, and the student’s total score is 50%, find the number of questions in the paper. By the given condition, 15+ 3 of (n-20) = ; 45+n-20 _ on 3 =a n+ 25 n 3 2 > 2n+50 = 3n sn = 50 2. There is a group of cows and chickens. ‘The number of legs was 14 more than twice the number of heads. Find the number of cows and chickens. Sample Questions and Solutions 61 Let the number of cows be = x and the number of chickens = y Therefore, total number of legs = 4x +2y ‘Total number of heads = 2+y Given :42+2y = 2%(c+y)+14 ie do+2y = 2x +2y+14 or = 7. Clearly y can be any whole number because it gets cancelled in the process. Therefore, No. of cows = 7 ; No. of chickens = any whole number. ‘The sum of four positive numbers is 680. If 5 is added to the first‘number, 5 is subtracted from the second, the third is multiplied by 5 and the positive square root of the fourth is extracted, we then get four equal numbers. What are the initial four numbers? Let the four numbers be p,q,7,8. Given: ptaqtrt+s = 680 — (1) pt5=q-5=5r = V5 (2) q=p+0; ra Pts. , By (1)&(2) 680 = p+q+rt+s ptptio) +P? + p45y 5p+ 5(p +10) + (p+5) + 5(p + 5)? 5 " = 680 x 5 = 5p? + Glp + 180. Solving we get p=20; g=30; r=5) - s=625 62 Mathematics Talent: Tests - Sub-Junior Level 4. Find all the two-digit numbers such that when they are divided by the sum of their digits, the quotient is 7 with no remainder. Let it be: * Value of number = 10c+y Its digit sum = x+y Given; W279 2 7 aty lz+y = T+Ty or3c = by ort = 2y But y cannot be equal to or more than 5 because in that case x will be equal to 10 or more thus giving (not a 2-digit but) a 3-digit number. ‘Therefore, y cannot be 1, 2, 3 or 4. We get x to be 2, 4, 6 or 8 correspondingly. Therefore, the numnbers needed are 21, 42, 63 and 84. 5. Two candles A and B of the same height are lighted at the same instant. A is consumed in 4 hours while B in 3 hours. Assume each candle burns at a constant rate. In how many hours after being lighted was A twice the height of B? Let the height of each candle = 1 unit. Let t be time required. heigh of A at time ¢ = 2x height of B at timet. =2-3 Sample Questions and Solutions 63 pot 4 t 1-j = meee a ned tees 8t—3¢ no} bt mol ee 2 wns =2hrs, 24 min. (5t/12) = 1 Therefore, £ = 12/5 hrs. = 2hrs, 24 min, 6. A man is due at a certain place at a certain time. If he walks at the rate of 6 kms per hour, he will be 15 minutes late; if he walks at the rate of 8 kms an hour, he will be 15 minutes early. Find the distance he has to walk. Let the distance = d kms. Let the time taken = t hrs. 1 1 d= (t+ 78 = (tps 6 8 G+ 7 = 8-5 68 ata = 2% 7 3 = % poe 4 distanced = (+ Ds nou ci et gal ES 64 Mathematics Talent Tests - Sub-Junior Level 7. A man loses one-third of his money; then he gains Rs.10. Then hhe loses one-third of his possession and again loses Rs.20. Now he finds that he has what exactly he had at the beginning. What was the amount he had originally? Let Rs a be the amount, the man had. He lost’ } of it. -. Remaining amount = 3 of it = Rs. jz. He gains now Rs.10. <. Now he has Rs. 22 +10. He again looses } of this. ’. Remaining amount = 3 of this amount. 2(2 =Rs5 (3 + 1») He again gains Rs.20. .. Now the amount would be Rs. 3 (3x + 10) + 20. This should be same as Rs. x 22 .2(Ze410) += 4 20 put t2 =. or 42 +60+180 = 92 ie, 240 =. 5¢ 12 = 48. He had Rs.48 initially. (Check the result). 8. For a class, copies of 9 maths books and 16 science books cost Rs.220. Each books costs a whole number of rupees. Find the cost of each maths book. Denote m for the cost of one maths book and s for the cost of one science book. Sample Questions and Solutions 65 9m + 6s Therefore, m Since m is a natural number, #>1 <6. But 41 is odd and a multiple of 9 and therefore 45+ can be 1, 3 or 5 only. 4851 = 5 is not possible 4s=1 = 1 is not possible 4851 = 3 is possible in which case s 3=7 gives m=24-12=12. Therefore, cost of maths book is Rs.12. 9. Write down your age in years. Multiply the number you have written by 10 and add 5. Multiply this sum by 10 again. Add the number of month in which you were born counting January as 1, February as 2 and so on. Subtract 50. The first two numerals on the left will be your age. The next two will be the number of month in which you were born. Explain how this works. Successive Operators _| Result ‘Suppose the age to be « | « Multiply by 10 102 Add 5 10 +5 Multiply by 10 100z + 50 ‘Add the month, say m_ | 1002 -+50-+m Subtract 50 1002 +m Now, m is a two-digit number and so the result follows. 10. On September 1, 2002, a mason was appointed by a contractor at Rs.150 per day, subject to the condition that whenever the mason was absent he would be fined at Rs.200 per day, as penalty, At the end of the month the mason found that he 66 Mathematics Talent Tests - Sub-Junior Level earned Rs.650 only. How many days did he work for? Let the number of days he worked be =. Therefore, no. of days he did not work = 30-2. Amount he received = Rs, 1502 Amount he paid as fine = Rs. 200(30 - 2) Therefore 150 x @ — 200(30 — x) = 650 150z — 6000 + 2002 = 650 + 3502 = 6650 ae ae S00 350 sz = 19, 11. Ina sports meet, one sportsman told another: “There are nine fewer of us here than twice the product of our total number?” How many sportsmen were there at the Meet? Final Test 2003 Solution: Let the total number be = z= y By the problem 10z+y+9=2cy. “. 2cy-y=102+9 (or) y(@z—1)=10r+9. y= Wee, z can take values from 0 to:9. Only when ¢ = 4 we get a whole number (=7). There were 47 sportsmen in the Meet. 12. The square of two consecutive positive integers differs by 2003, What is the sum of these two integers? Final Test 2003 Solution: Let the integers be 2,2 +1. Sample Questions and Solutions Given (x +1)?—2? =. 200: 2441-2? = 200: Qe+1 = 200 a+(x+1) = 200: ie. required sum = 200 13. Given 2,y are positive prime numbers. If find y. 42? = ‘Therefore 2” is odd and so « is odd. lets = Therefore 2? = (2k+1)? = 4+ 4k+1 = ay? = = y? is even. 67 13 3 3. 13 3. 2? —2y? = yr +1 +1 4k? + 4k +1 Qy? +1 4k? + 4k So, y is even. y is also prime. Therefore, y = 2. 14. Solve for real values of ,y given that Qa? + y? + 2ey —40+4=0. Only one equation is given but tnere are tw small trick helps us to solve this. Qe? ty? + %ey—4o+4 = a+y+2eyt+a?-40+4 = (c+y)P?+(e-2)? = The sum of the squares of two quanities is each of them is zero. 0 unknowns. A zero only when 68 Mathematics Talent Tests - Sub-Junior Level (ie.,) If a,b are real and a? +6? =0, then a=0,b=0. Thus we have here, s+y=0 and c—2=0 | g=2and y=-2. @+P+ce PO + Pe + Pa? a? +b? +02 + 2ab + 2be + 2ca = (a +b +c)? =0 here 15. If (a+ b+ c) =Othen find a+b +c? = —(2ab+ 2bc+ 2ca) = -2(ab+ be + ca) a+ Pace [-2(ab + be + ca)]? ORF Perea — (ab + be + Pa?) [4(ab + bc + ca)?] 02? + Pe + 2a?) [4(a20? + bc? + ca? + 2ab2° + 2abc” + 2a7bc] [a2b? + b2c? + cPa?) [4(a26? + bc? + ca? + 2abe(a +b + ¢)] [a?0? + Pe + a7] (4(a28? + be? + c?a)] [a?0? + Bc? + Pa] = 4. 16. A= {a,b,¢,d,e} is a set of five integers. We take two out of the numbers in A and add. The following ten sums are obtained 0,6,11, 12, 17, 20, 23, 26, 32, 37. Find the five integers in the set A. Solution: We may assume that a ZCBP = 90° — 60° = 30° A PCB is Isosceles and PB = BC. ZPCB = ZCPB = = 759. > o [Note: A, P,C] are not collinear. Check! <. Answer is (D) 8. The area of the largest possible square inscribed in a circle of unit radius (i.e. radius=1) is A)2 B) 2 ©) 4 D) 2v2a Diagonal of the square = Diameter of the circle=2 Therefore, area of square 5x (diagonal )? 1 2 x4 2 sq.units .. Answer is (A) 9. A rhombus has one diagonal double the other. If the area of rhombus is kem? then the length of its side is A) S¥fcm B) Eom —C) Fem dD) YS 4 Mathematics Talent Tests - Sub-Junior Level Area of rhombus = }x product of diagonals. 1 2 = 5 (4a) x (2x) (say) = 49°. Given 427 =k. «*, foa= ¥E, If side is a,a? = (22)? +2? = 52?. (Because diagonals cut at rt angles each of the 4 triangles in the figure is rt angled] <. Answer is (C). 10. The adjacent sides of the decagon shown meet at right angles. The perimeter of the decagon is (A) 28 (B) 192 (0) 56 (D) 36 Look how the diagram given can be transformed into an equivalent, diagram of the same perimeter as follows, gradually. 12 v ~T 1 1 ol | | .: The perimeter 2(12 + 16) = 56. . Answer is (C). Sample Questions and Solutions 7 11. The adjoining figure is formed by two squares. The side of each square is a whole number. If the area of the figure is 58 square centi-meters, its perimeter is A) 58cm —-B) 34cm C) 29cm —D) 16cm Screening Test 2002 Solution: The square numbers are 1, 4, 9, 16, 25, 36, 49, 64, - Because (i) the area = sum of two squares and (ii) each square is a whole number, 58 =a? +y?=49+9 is the only possibility. z=7 and y=3. Perimeter = 32 +(x—y) +3y = 40+2y = 28+6em = 34cm. c. Answer is (B). 12. A rectangle with perimetre 44 is partitioned into 5 congruent rectangles, as indicated in the diagram. The perimeter of each of the congruent rectangle is A)10 B)20 C)24 D)48 Screening Test 2003 76 Mathematics Talent Tests - Sub-Junior Level Solution: Let the dimensions of the augment rectangles be a,b respectively as indicated in the figure. Given perimeter of bigger rectangle = 44 * ie 4a4+5b = 44 (1) Also 2a = 36(look at the figure!) => 22-3 = 0 (2) Solving (1) & (2), b=4 and a=6 .', Perimeter of smaller rectangle = 2a + 2b = 12+8 = 20 .. Answer is (B) 13. A rectangle is divided into four smaller rectangles, as shown in the figure. Three of them have areas 6,18 and 36, as indicated. The area of the shaded rectangle is A) 30 B) 24 C)12 D)18 Screening Test 2003, Solution: Let the dimensions be a,b,c,d for the rectangles as indicated in the figure > ac = 36 (Q) ad = 18 (2) oder 6 (3) Sample Questions and Solutions 7 from (1),(2) & (3), ac x ad x bd = 36x 18x6 (or) adxadxbe = 36x 18x6 ke ae pee ; = ai a YW. =Bx x6 = 2x6=12, . Answer is (C) 14. The small square is surrounded eee by four congrement rectangles as shown, to form a large square. If the perimeter of each rectangle is 16, the area of the large square is Prec A) 64 B) 256 C) 32D) 224 Let the length, breadth of each rectangle be x,y respectively. Then Qc+2y = 16 ty = 8 ~. Area of large square = (+y)(z+y) = (c+y)? = 8 = 64 o. Answer is (A) 15. The area of the quadrilateral shown 2 in the figure is ; A) 6 B) 2+ V6 P C) 24+V8 D) viz Screening Test 2002 78 Mathematics Talent Tests - Sub-Junior Level Solution: From AADC AC = J? +2? = 2V2 4 2 Since AABC is isosceles and b BOLAG, 8 Cc AO = 00 = V2, From ABOC, BO = /BC?— 0G? = V6=2 =2 . Area of quadrilateral ABCD = area of AABC + area of AADC = (jx Ae = 20) + (5x 4p x0) = (Lxovixe2)+(tx2x2 ee) 2 = 242V2=2v8. 1. Answer is (C). 16. Ifall the diagonals of a regular hexagon are drawn, the number of points of intersection, not counting the corners of the hexagon is (a) 6 (B) 13 ()7 (D) 12 Screening Test 2004 Solution: Let ABCDEF be the regular hexagon. Then there are nine diagonals - 3 diagonals from each of three alternate vertices: AC, AD, AE, CE, CF,CA, EA,EB and EC. From the figure it is easily seen that the Sample Questions and Solutions 79 number of points of intersection is 13. .. Answer is (B) Note: The diagonals AD,BE,CF are concurrent at the centre O of the regular hexagon. 17. The area of the shaded region in the diagram is (A) 9 (B) 3v2 (C)18 — (D) 6V3—3v2 Screening Test 2004 Solution: The shaded area is the area of the hexagon ABCDEF minus the area of the square PQRS whose diagonals are both equal to 3 units. Thus the required area —_—— is equal to 7(6 +3) -§ + 5(6 +3) - 3 — 338 we $=9 square units. ©. Answer is (A) Note: The hexagon ABCDEF is not a regular hexagon and is symmetric about PR and FSQC. 18. In the adjacent figure ACD is a line. The measure of angle BCE is B A) 120° B) 150° ©) 210° =D) 270° From Rt ADCE,« +22 = 180 - 90 = 90 80 Mathematics Talent Tests - Sub-Junior Level -. 32 = 90° or z° = 30°. In AABC,mZBCA = 180° — 22° — 22° 180° — 42° = 180° — 120° = 60° mZACE = 180° — ZDCE = 180° — 90° = 90° :, mLBCE = mZBCA + LACE = 60° + 90° = 150° 19. The altitude drawn to the base of an isosceles triangle is of length 8 and the perimeter is 32. The area of the triangle is A) 32 B) 40 c) 48 D) 56 Screening Test 2002 Solution: Let 2 = length of equal side. Then base = 32 — 22. By Pythagoras Theorem 2? = 8? + (16 — x)? = 64 + 256 — 322 + 2? .. 322. = 320 or z= 10. Base = 32-2¢=32-20=12, Area = ; x base x height = 5xix8=48, L am -:. Answer is (C). a 20. In the adjacent figure DE bisects ZBDA and AD bisects ZBAC. ‘The measure of ZBED is A) 85° B) 95° C) 75° D) 105° - Screening Test 2002 2) 22. Sample Questions and Solutions 81 Solution: From AABC, ZA+ZB-+ ZC = 180° ie.,2y+ 40° + 80° = 180° > y= 30°. In AADG, external ZADB = y + 80°. J. 2x = 30° +80° = 110°. 1 DS 5B. From AADE, external BED = x + y = 55° + 30° = 85°. ~. Answer is (A). In a right angled triangle, if the square of the hypotenuse is twice the product of the other two sides, then one of the angles of the triangle is A) 30° B) 45° ©) 60° D) 15° Screening Test 2002 Solution: By Pythagoras, Dey tech = 2zy, (given). 2 -Isyt+y? = 0 or(z—-y)? = 0. a-y = Oorz=y. +, It is an isosceles right angled triangle. Each one of the other two acute angles should be 45°, +, Answer is (B). A circle is added to the equally ce ewe spaced grid alongside. The largest we eee number of dots that the circle can ry pass through is eee ee (A)4 (B)6 (C)18 (D) 10 ee eee Screening Test 2004 23. 24. 82 Mathematics Talent Tests - Sub-Junior Level Solution: Denoting the dots in the first, second, third, fourth and fifth rows respectively by A1,A2, As, Aa, As; Bi, Bo, Bg, Ba, Bs; C1,C2,C3, C1,C5; Di, D2, Da, Da, Ds; and E},E,E3, Es,E5, the circle with centre C3 and radius C3A2 passes through the eight points Ap, A4, Bi, Bs,D,Ds,E2 and E,. This is the maximum number of points. None of the options given is 8. Note: Perhaps option (C) viz., 18, is a misprint for 8. With this correction the answer will be (C). D,A,O,B and C are points on a line. Circles are drawn with O,A and B as centres as shown. If the radius of the smallest circle is 1 cm, then the length CD is A)3cm BB) dem aa oa C)5cm =D) 6m AB is the diameter of the smaller circle. AB is the radi of the bigger circles centred of A, B. <. Length of radius DA = AB = radius BC. ©. Length of CD = 2cm+2cm+2cm = bom Note: AB =2x OB =2x lem = 2cm Answer is (D) In the triangle ABC, D is a point on the line segment BC such that AD = BD =CD. The measure of angle BAC is (A) 60° (B) 75° (0) 190° (D) 120° Screening Test:-2004 25. Sample Questions and Solutions 83 Solution: Since DA = DB = DC, D is the circumcentre of AABC and BC is the diameter of the circumcircle. So, mZBAC = 90°. None of the options is 90°. Note: (C) should have been 90° and not 190°. No angle of a triangle can be greater than or equal to 180°. Points P,Q,R and S divide the D c sides of a rectangle ABCD in -S the ratio 1:2, as shown in the figure. What fraction of area of the rectangle is the area of the A parallelogram PQRS? P B 2 3 4 5 a? B) 2 o4 p) é Screening Test 2003 y Solution: Ra a s 2a 2 Q a 4 dp 2a B Let AP=QB=CR=DS=a and PB = QC = DR = AS = 2a units respectively. Area of square ABCD = (3a)? = 9a? sq. units Area of 1l'gm PQRS = Area of square — Area of 4 augment rt A ‘les 1 = 90? — 4x (5 xa x 2a) 84 Mathematics Talent Tests - Sub-Junior Level = 50°Sq. units = 3 x (94?)Sq, units ; x Area of square ABCD -. Answer is (D) 26. A cylindrical pail containing water drains into a cylindrical tub of diameter 80 cm and height 100 cm, while resting at an angle of 45° to the horizontal, as shown. How deep is the water in the tub when its level reaches the pail? A) 40cm B) 50cm C) 60cm D) 45cm Screening Test 2002 Solution: Take points A,B,C as in the figure. By symmetry, AABC is isosceles right angled. Draw ADLBG. Then AADB is also isosceles right angled. <. BD = DA(= 40). +, Depth of water required = 100 — 40 = 60cm. , Answer is (C). 27. Turning a screw driver by 90°, we can drive a screw 5 mm deeper into a piece of wood. How many complete revolutions are needed to drive the screw 6 cm into wood? A)2 B)3 0) 4 D) 12 Screening Test 2003 28. Sample Questions and Solutions 85 Solution: Turning the screw driver by 1 revolution (ie 360°) penetrates the wood by 4.x 5mm ie, 20mm. .“.3 revolutions are needed to penetrate 6cm i.e 60mm. <. Answer is (B) Suppose we cut out an equilateral triangle and fold its vertices to meet at the centre. The folded figure has the shape of A) a square B) an equilateral triangle C) a regular hexagon D) a rhombus . Answer is (C). See figure for explanation. 29. The diagram shows a ‘net’ to make a cube. When 30. the cube is made, the face not adjacent to E is A)A B)B B oc D)F a/O| > 0 ‘Not adjacent? actually implies ‘opposite to’ here, So answer is (C). (‘Try to make a cube) Observe the way the squared paper in the figure is divided into three blocks. Which of them is a net for an open cubical box? A) X only B) ¥ only ae is shaded ©) ¥ and Z only D) All ¥ block is shaded 7 Screening Test 2002 7 Plock is shaded (7) 86 Mathematics Talent Tests - Sub-Junior Level Solution: Each one of these can be folded into an open cube. Amawer ie (D). ay oy ate A cube of side 6 cms is cut into a number of cubes, each of side 2 cms. The number of cubes will be A) 27 B)9 C6 D)3 No. of cubes = —Wottme‘sr tack bean” = Seoee = 27 ©. Answer is (A). 31. 32. The larger cube in the figure consists of 27 unit cubes. If all the six faces are painted in colour (say, in blue), how many unit cubes have only one (blue) coloured face? A) 6 B) 8 ©) 12 D) 27 Screening Test 2002 Solution: By careful analysis of the figure, it is easy to see that the answer is 6. [In fact, for an nxnxn cube, the number of unit cubes with one coloured face will be 6(n — 2)?]. ., Answer is (A). 33. In the diagram, there are four lines. The line whose graph could be y = 10—¢ Screening Test 2002 Sample Questions and Solutions 87 Solution: y= 10-2 is a line with negative slope (—1). «. It can be [3 only. <. Answer is (C). 34, Suppose a mirror stands on the Y- axis drawn ona graph sheet. Then, = 1 pea, the reflection of the point (-2,4) in the mirror will be 3 fel A) (2,4) B) (2,-4) C) (4,2) D) (4,2) Imagine that there is a mirror along y axis. Then (-2,4) will be reflected to (2,4). So the answer is (A). PART-II 1. A rectangle has sides of integer lengths (in cm) and an area of 36 cm?. What is the maximum possible perimeter of the rectangle? Final Test 2002 Solution: Area = 36cm?. Sides have integer lengths. . We make a list of possibilities Side lengths Perimeter 1,36 -2(1 +36) = 74 2,18 (2+ 18) = 40 3,12 (3+ 12) = 30 4,9 2(4 +9) = 26 6,6 2(6 +6) = 24 ., The maximum possible perimeter is 74 cm. 2 3. 88 Mathematics Talent Tests - Sub-Junior Level Identical isosceles right triangles are removed from opposite corners of a square resulting in a rectangle. If the sum of the areas of the cut-off pieces is 450, find the length of the diagonal of the rectangle. Final Test 2003 Solution: From the figure xvi, b = yV2(45° — 45° — 90° triangles). a Sum of the areas of the four triangles () ++) =) +) = Floreat ao 40%) 7 302 + 2a? + 2y? + 2y2) = 22? + 29? which is given as 450. Length of diagonal of the rectangle = /2x 450 = 900 = 30. From a square metal plate, a circle of maximum size is cut out; again from this circular plate a square of maximum size is cut out. Find the ratio of the metal wasted to the metal of the original square. Area of original square ©. diameter of circle = a , diagonal of the smaller square = a Sample Questions and Solutions 89 Wasted metal wv required ratio = % : : 12 4, Ina trapezium ABCD, ABI|CD and 2D =2ZB. If DO =p and AD =q find AB. Solution: Draw DE||CB to A c meet AB at B. EBCD isa lle and so 2B=ZEDC = . ZADE = 2 & e B since whole angle ADC = 2c (given). Also ZAED =z (alternate angle of ZCDE). », ZADE = ZAED AB = AD=q °° Also BB = DO=q AB = AE+EB=p+q 5. The sum of the lengths of the three sides of a right triangle is 18. The sum of the squares of the lengths of the three sides is 128. Find the area of the triangle. Final Test 2002 Solution: a+ Pac? (1) b m a+b+c=18 (2) a+ +07 = 128 (3) 90 Mathematics Talent Tests - Sub-Junior Level By (1) and (3), 2c? = 128 which gives c? = 64 and hence c=8, 2. (1) > a? +8 = 64 (4) “. (2) a+b=10 (5) os Bab = (a +)? — a? — = 100 - 64 = 36 ©. Area of the triangle = ; x18 =9. In fact, a= 5+ V7,b=5-V7 and c=8. ABC is an equilateral triangle (see figure). D is some point on BC. If DE =3 and DF =7 find the length of altitude form A to BC. Let AB=BC=CA=x. AABC = AADB+ AADC 1 1 1 Lexum = (een) + (Ixex1) AH = 34+7 = 10 . In triangle ABC, we are given that ZA = 90°. Median ‘AM, angle bisector AK and the altitude AH are drawn. Prove that ZMAK = ZKAH Final Test 2004 Solution: We have AM = MB = MC since ZA = 90°. If ZMAK = 2°, then M ZBAM = 45° — 2°. From the isosceles triangle AMB B we get ZABM = ZABC = 45° — 2°. «. LACB = 90° — ZABC = 90° - (48° - «' =45°+2°. Sample Questions and Solutions 91 From the right angled triangle AHC we get ZCAH = 45° — z° andhence 2K AH = ZCAK-ZCAH = 45°-(45°—2°) = a°. Thus 2MAK = ZKAH. 8, An equilateral triangle is drawn inside an isosceles triangle, as shown in the figure. Show that 2 is the arithmetic mean of y and z. ZB = 180° — x — (180° — 60° — y) ZC = 180° — z — (180° — 60° — 2) Since 2B = ZC, equating the two above 180° — x — (180° — 60° ~ y) = 180° — z — (180° — 60° — z) This simplifies to 4 ytz=2r which is required. 3 c 9. Consider the collection C' of all isosceles triangles of area 48 sq.units, whose bases and heights are integers. How many triangles are there in C? How many triangles in C have their equal sides also of integral lengths? : Final Test 2004 Solution: If we use the formula, Area of a triangle == base x height we get base x height = b x h=96. So the various possible values for (b,h) are (1,96), (2,48), (3,32), (4,24), (6,16), (8, 12) (12,8), (16,6), (24,4), (32,3), (48,2), (96,1). For each of the above 12 choices for (bh) we.have an isosceles triangle satisfying our requirements. So, there are 92 Mathematics Talent Tests - Sub-Junior Level 12 such triangles. Of these only two triangles have sides of integral lengths. They ‘are got when b = 12, h = 8 and b= 16, h=6 as 6?+8? = 10? and 8?+6? = 107. Thus the triangles with integral sides have sides of lengths 12, 10,10 or 16, 10,10. 10. Let ABC be an acute angled triangle with AD, BE, OF as the altitudes (i., D is the foot of the perpendicular from A on BC and so on...). If the altitudes meet at the point O, find the measure of the angles ZBOC, rs ZOOA,ZAOB intermsot of the angles 2A,2B,20 of f/f) > the triangle ABC. Final Test 2004 Solution: From the right triangles BEC and CFB it is clear that ZCBE =90° — ZC and ZBCF = 90° - ZB. Hence in ABOC we have ZBOC = 180° - (LCBO + ZBCO) = 180° — (90° - ZC +90° - 2B) = 2B+2C Similarly we find ZCOA and ZAOB. Thus we have Z2BOC = £B+2C; ZCOA=2C + 2A; ZAOB=2A4+ ZB 11. Ina triangle PQR, Sis the mid point of PQ and PR > QR. Prove that PSR is obtuse. Since PR > QR we have Q > P. If we prove that the foot of the perpendicular from R to QP Q KS P lies to the left of S, then PSR will be obtuse. 13. Sample Questions and Solutions 93 Draw a line QP! through Q such that P!QP = P. If P'S is joined clearly it is the perpendicular to P. Since R lies on the left of P', the perpendicular from R to PQ also lies on the left of Pls. PSR=90+KRS. .. PSR is obtuse. . In the figure ABCD and AEFC are squares. AABG is a right triangle. If AABG has an area }, find the area of the polygon AEFCD. Final Test 2003 Solution: AABC is isosceles; Let AG =GC =x. Then BGO =a A, 2 }xgtaboato? x] .. a= (= AG) E a 1 AC =2x B= V3. 1 Area of square AEFC = (v3)? = 2. e Hence the area of the polygon AEFCD = area of square AEFC~ area AADC =2-}=3. The areas of the six sides of a closed rectangular box are (in m2) 48, 80, 60, 48, 80, and 60. Find its volume. Let 1,b,h be the length, breadth and height of the box. Given 1b = 48, bh = 80, hl = 60 Ibx bh x hl = 48 x 80 x 60 fie xb? xh? W (4x 4x3) x (4x 4x 5) x (4x 3x 5) 4° x 3 x 5? 210 x 3? x 57, W Taking sqrt. lbh = 2° x3 x 5 = 480(cm3). 14. 15. 94 Mathematics Talent Tests - Sub-Junior Level 42 identical cubes, each with 1cm edge are glued together to form a cuboid. If the perimeter of the base of the cuboid is 18cm. Find the height of the cuboid. Final Test 2003 Solution: Perimeter of the base of the cuboid = 18cm Length + breadth of the base = 19cm Each edge of every cube measures = 1cm Base of the cuboid could be (1 and 8) or (2 and 7) or (3 and 6) or (4 and 5). ‘There are 42 cubes in all. The volume of the cuboid = 42 x 1x 1x 1 cm’. The possibilities to be considered are: Length | 1] 2 Breadth | 8] 7| 6| 5 Volume | 42 | 42 | 42 | 42 Of these only 2 x 7 base yields a height which would be a multiple of 1 em (42 +14 = 3). ., The height of the cuboid is 3 cm. Triangle ABC is divided into four regions with areas as shown in the diagram. Fig: Sample Questions and Solutions 95 Solution: Let us call the point of intersection of BE and CF inside the triangle as D. Let area @ and ares am a_AAFD _AF _ AAFC _a+b+6 9” BBFD~ FB™ ABFC” 9+11 © Then we have a_a+b+6_-a_ b+6 == 3 = = 1 Rheeis aici. cing-eani cig ial @ Similarly, & - S4ED _ AB _AABB _a+b+9 ymiar'ys §~ ROED EC ACEB 6+11 ~ Therefore b_a+b+9 b_a+9 a7 6401 6" i @) From equations (1) and (2) w get Ma -9b-54=0 (3) 6a —11b+54=0 (4) Solving the system (3),(4) we get _ 1080, 918 oer eT" Hence mat b= 1098 saath, . A circle is inscribed in a rhombus, one of whose angles is 60°. Find the ratio of area of the rhombus to the area of the inscribed circle. 96 Mathematics Talent Tests - Sub-Junior Level Solution: We need only the A ratio. Without loss of generality jf we assume that the side length of the rhombus=2units. Draw a perpendicular AM from the top corner of the rhombus to the base. We get a 30°—60°—90° ds A triangle. Dd M h=V3. Area of rhombus = 2V3. 4 Diameter of circle = height h. .”. Radius of circle = $. 2 o, Area of circle = «x (2) ae 4° 3 a ~. Ratio required = 2V3 : * = 8V3 : 30 =8:2Vv3. 17. ABCD is a cyclic quadrilateral (which means that a circle passes through the vertices A,B,C, D).In other words the vertices A,B,C,D, in that order, lie on a circle. If the diagonals AC and BD cut at right angles at E, prove that AE? + BE? + CE? + DE* = 4R? where R is the radius of the circle ABCD. Final Test 2004 Solution: Let O be the centre of the circumcircle of the quadrilateral ABCD. It can be easily observed that ZAOD + ZBOC =22ABE +2ZEAB = 2(ZABE + ZEAB) = 180°. Sample Questions and Solutions 97 This gives 2A’OC = 180° — ZBOC = ZAOD where BO meets the circle again in A’: Therefore AA'OC = AAOD (SAS property). AD = A'C AE? + BE? + CE? + DE? = AD? + BC? = A'C? + BC? = A'B?(:: ZA'CB = 90°) =4R. 18. About how many lines can one rotate a regular hexagon through some angle 2°(0° < 2° < 360°), so that the hexagon returns to its original position? Final Test 2002 Solution: The Axes of rotational symmetry are: i) 3 lines joining opposite vertices ii) 3-lines joining the mid points of opposite sides. iii) 1 line through the centre and perpendicular to the plane of the hexagon. ©. The answer is 7_ 19. Consider a cylinder of height 4cm and the perimeter of the base circle 3em. P isa point on the lower rim and Q is the vertically point above P on the upper rim, A thread is wound once round the cylinder starting at P and ending at Q what is the length of the thread? It will be surprising to. note that this problem involves Pythagoras theorem. Cut the cylinder along the vertical 98 Mathematics Talent Tests - Sub-Junior Level line PQ and open it up. We get a rectangle. It is clearly visualized that P,Q are the opposite vertices of this rectangle. The thread is nothing but the diagonal PQ PQ = VB FE = Bem 20. A child has at its disposal n small wooden cubes, all the same size. With thera he tries to build the largest cube he can, but discovers that he is short by exactly one single row of small cubes that would have formed an edge of the large cube. Prove that n is a multiple of 6. Let a = side of the largest cube to be made. Then by the given condition. n = a-a = a(a?—1) = a(a+1)(a-1) = (a-1)xax(a+l) product of 3 consecutive integers a multiple of 6 . 21. Given that a? — 6? = 105 and a and b are two relatively prime positive integers ( two positive integers m and n are relatively prime if their g.c.d (m,n) = 1), find all such a and b. After having found all such a and 6, if one draws a triangle ABC with sides having lengths a?—b?, a? +0? and 2ab find the area of all such triangles. Final Test 2004 Solution: It is given that a? —B = (a+b)(a—b) =1x3X5x7= 105. So, the various possibilities for the ordered pair (a — b,a +6) Sample Questions and Solutions 99 are (a — ba +b) = (1,105) (a-b,a +6) = (3,35) (a—b,a+b) = (5,21) * (a—b,a +6) = (7,15) Solving we get (a,b) = (53,52), (a,b) = (19, 16), (a,b) = (13,8), (a,d) = (11,4) The triangle with sides a? +, a? —b?, 2ab is always a right angled triangle as (a? — 87)? + (2ab)? = (a? +.82)?, The area of such a triangle is ab(a? — 62) = 105ab. Thus the required areas are 289380, 31920, 10920 4620. CHAPTER - VIII LOGICAL ARGUMENTS PART-1I 1. Rahim wants to arrange a party of a certain number of people such that no two of participants (whose dates of birth he knows) of the party will have birthdays in the same month. The number of people to be invited for the party cannot exceed A)12 B)13 C) 364——iD) 23 Screening Test 2003 Solution: There are 12 months in a year. If at all, all the invitees to the party are born on different months, can be a maximum of 12. The 13 invitee, if any, should be born on a month which belongs to any month of the same month then. w. Answer is (A) 2. Today is Saturday. What day of the week is 100 days from now? A) Friday B) Tuesday C) Monday) Sunday Screening Test 2003 Solution: The days of the week are cyclic. satay Friday ‘Sunday Monday ‘Thursday Tuesday Wednesday p= 100 Sample Questions and Solutions 101 100 days = 14 cyclic weeks + 2 days. Ifa cycle starts from Sunday, it ends on Saturday. (Note: 1 day from Saturday is a Sunday) ©. 100 days from Saturday is equivalent to 14 cyclic weeks (ends on Saturday) and two days forward. (i.e. Monday]. ©. Answer is (C) 3. If 15! January 1997 was a Wednesday, then 15" February 2006 will be a A) Wednesday B) Tuesday C) Sunday D) Friday Screening Test 2002 Solution: Let us calculate the total period of days upto 14** Feb. 2006, Denote a week by ‘w’ and days by ‘d’. Total period = Some weeks + 1ddays = multiple of7 days (ie., full weeks), +, The next 15 day of Feb. 2006 will be a Wednesday. +. Answer is (A). 4, One hundred and twenty students take an examination which is marked out of a total 100 (with no fractional marks). No three students are awarded the same mark. The smallest possible number of pairs of students who are awarded the same rank is ) B) 10 C) 20 D) 19 Screening Test 2003 102 Mathematics Talent Tests - Sub-Junior Level Solution: The range of score marks will be from 0 to 100 (in - whole numbers). .. The number of students who can score different marks is a maximum 101. But there are one hundred and twenty students. The remaining 19 students must score marks that are already scored by any one of the 101 students. Since no three students score the same mark, only 2 students could have scored the same mark. . The smallest possible number of students in pairs who are awarded the same rank (or scored the same mark) is 19. ©. Answer is (D) 5. The cost of a single two-litre bottle of juice is 5 times the cost of 1 cup of juice. Ifa single two-litre bottle of juice costs Rs.6 more than 3 cups of juice, how much does a single two-litre bottle of juice cost? A) Rs.10 B) Rs.12 C) Rs.15 D) Rs.18 Screening Test 2003 Solution: Let’s “analyse the problem by pictorial approach. . Lcup of Juice costs Rs.3 (or) 5 cups of Juice costs Rs.15 (oz) single 2-litre bottle of Juice costs Rs.15. . Answer is (C) 6. Which one of the following is true? (i) If A isa brother of B, then B is a brother of A Sample Questions and Solutions 103 (ii) If A likes B and B likes C, then A likes C (iii) If A# B, and BAC, then AAC A) Both (i) and (ii) B) (ii) only B):C) All the three D) None of the three If A isa brother of B, then B may bea sister of A. So (i) is false. A may like B and B may like C, but then there is no guarantee that A will like C. So (ii) is also false. 2 then we see is also false. Suppose for example A = 2,B = 3 and C that A#B and B#C but A=C. So (ii ‘Thus (i), (ii) and (iii) are not always true. . Answer is (D) . For an integer n, a student states the following: I. If n is odd, (n +1)? is even IL. If n is even, (n—1)? is odd IIL If n is even, /(n—1) is irrational. Which of the above would be true? A) 1&1 B) I&II C) ALL D) W& I Screening Test 2002 Solution: n is odd > (n+ 1) is even + (n+1)? iseven. +, I is ‘True. n is even = (n—1) is odd = (n—1)? is odd. o. ID is True. n is even need not imply “i= is irrational”. For example, 10 is even but Y10—1 = V9 is rational. ©. IIL is False, ©. Answer is (B). 104 Mathematics Talent Tests - Sub-Junior Level 8. The adjacent table defines an operation x. For example, from the table we find * o axc=dandbxd=b. If bxa=a, then 2x2 is A)a B)b Che Dd elole lalla alolole» fa alolele elaololols cle lato bxa=a2=c, since by table bxse=a. ©. @=cxc=b from the table. ©. Answer is (B). 9. Ina magic square, each row and each column and both main diagonals have the same total. ‘The number that should replace x in this partially completed magic square is (A) more information needed (B)9 (C)10 (D) 12 Screening Test 2004 3 ale 15 Solution: From the first column, the total of the magic square is 18 +2. Therefore the number in the central cell is 18+2—(15 +5) =2—2 This implies that the last number in the first row is 18 +2 —(c+—-2) = 20~c, To find the number in the third cell of the third row we can use the third column or the diagonal through it. Hence this number is equal to 18 +2 —(13-+2—2) or (18 +2) —(20-2+18). Thus 7=22-17 or c=12. ©. Answer is (D) 13, 9|8 5 | 10] 15 1L| 7 Note: The completed magic square will be as | 12 in the adjoining table. Sample Questions and Solutions 105 10. The image of INMO when reflected in a mirror is (A) IMWO (B) OMNI (C) INWO__—(D) OWNI Screening Test 2004 Solution: If INMO is written on a transparent paper and held in front of a mirror the reflection reads as OMWI. This is not any one of the options given. On the other hand, if the paper with INMO written is held in front, its reflection in a horizontal mirror below the paper will be TMW 0’. “s. Answer is (A) 11, Nine dots are arranged such that they are equally spaced horizontally and vertically as in the figure. The number of triangles which are not right angled triangles that can be formed ° * ° with the above dots as vertices is (A) 18 (B) 21 (c) 40 (D) 24 Screening Test 2004 Solution: Dots belong to the same row or the same column or the same diagonal do not form a triangle. Name the dots as A, B,C,D,E,F,G,H,I as shown Taking A,B of the first row and joining A p them to the six points of the second and. * © © third rows, we can get six triangles. Of © © » these .AABD,AABE,AAGB and AABH PE F are right angled. Hence only AABF GH I eters and AABI satisfy the given condition. Similarly with B,C we have ABDC and ABGC. With C,A we have only AAHC, since AAEC is Tight angled. (Note that if AB = BC =a, then AC? = 402 and AE? = CE? = 2a? s0 that AC? = AB? + CE). 106 Mathematics Talent Tests - Sub-Junior Leel Thus every row, except the second, contributes 5 triangles. Similarly every column except the second will contribute 5 triangles. The second row and the second column will give only 4. Thus on the whole we get 28 triangles. Besides thses there are four more AAFH,ADCH,AIBD and AGBF. (These triangles have a vertex from each row and also form each column). Thus the total number of non-right-angled triangles that can be formed is 32. Thus none of the answers given is correct. Answer: 32 PART-II Rengu and Dingu are friends. One of them lies on Mondays, Tuesdays and Wednesdays and tells the truth on the other days of the week. The other fellow lies on Thursdays, Fridays and Saturdays and tells the truth on the other days of the week. On a particular noon, the two had the following conversation: Rengu: I lie on Saturdays Dingu: I will lie tomorrow Rengu: I lie on Sundays On which day of the week did this conversation take place? : Final Test 2002 Solution: Clearly Rengu is lying and is the one who tells the truth on Saturdays. The conversation should be on a Monday, Tuesday or Wednesday. => Rengu’s statement is true only on Wednesday. =, It is Wednesday. Sample Questions and Solutions 107 2, Three counters A,B and C are coloured with three different colours red, blue and white. Of the following statements only one is true. 1, A is red. 2. B is not red. 3. C. is not blue. ‘What .is the colour of each counter? Final Test 2004 Solution: We use the notation (R,B,W) to denote that A is coloured red, B is coloured blue and C is coloured white. With this notation (B,R,W) will mean that A is coloured blue, B is coloured red and C’ is coloured white. @ 1) |o (R,B,W) | True | True | True (R,W.B) | True | True | False (B,R,W) | False | False | True (BWR) | False | True | True (WRB) | False | False | False (W,B,R) | False | True | True From the table it is clear that A is coloured BLUE, B is coloured RED and C is coloured WHITE. 3. Out of 30 people, 24 like coffee, 18 like tea and 10 like fruit juice. If none of them like all the three, but four like coffee and fruit juice and four more like tea and fruit juice, how many like coffee and tea? We can find the answer by trial and error, but a Venn diagram can make things easy. By the diagram 10 + (24 — 4) + (18 — 4) = 44 But this includes the people in the shaded region as C'’s and T's, 108 Mathematics Talent Tests - Sub-Junior Level ¢ 24 An 10 F Therefore, the number in the shaded region = 44 - 30 = 14. «, Answer is 14, 4, Isaw 9 men gather in a room. Each one of them shake hands with each of the other ones, How many handshakes did I count in all? (I did not shake hands with anyone; I was only an observer). Let the person be A,B,C,D,E,F,G,H,I. Let us list the handshake. AB AC, BC AD,BD,CD AI, BI,CI, DI, Total = 36. Another Method: A will shake hands witli 8 persons. B will shake hands with 7 persons. (because he has already shaken hands with A) G will shake with 6 persons «+. and so on. Total handshakes = 8+7+6+5+4+3+2+1 = 36. Sample Questions and Solutions 109 5. A triangle with sides of integral lengths 2, 3, 4 is such that its perimeter is less than 13. How many more such triangles (with this condition of integral sides having perimeter less than 13) can you find? Let the sides be a,b, where a,b,c are integers. Given: a+b+e< 13 ‘Any two sides of a triangle when added will be greater than the third side. ‘Therefore, c< a+b Because c is the largest side, c > 4. Therefore, if c= 4, we can have a = 3,5 =2. If c= 5, we can have a=4,b=3 or a=4,b=2. Therefore, all possibilities are (2,3, 4), (2, 4, 5), (3, 4,5) => Including the triangle given in the sum, there are three such triangles. 1. 2. CHAPTER — Ix NUMBER THEORY PART-I A certain number has exactly eight factors including 1 and itself, Two of its factors are 21 and 35. The number is (A) 105 (B) 210 (©) 420 (D) 525 Screening Test 2004 Solution: Since the number has exactly eight factors, it has the representation, p{,p}p2 or pipop3 where p’s are primes. (For example, if it is pipz then the factors are 1,P1,PiPt, P2,Pip2, Pipa and piper.) As 21 and 35 are its factors 3,5 and 7 are its factors. Hence the number is 3 x 5 x 7 = 105. . Answer is (A) Note: The eight factors are 1,3,5,7,15,21,35 and 105. ‘The largest positive integer which cannot be written in the form 5m+7n where m and n are positive integers is (A) 25 (B) 35 (C) greater than 100 (D) greater than 350 Screening Test 2004 Solution: The number 26 can be written as 5x 1+7x 3. Hence (A) is not the answer. 33 can also be written as 5x1+7x4. Also 34 can be written as 5x 44+7%x 2. Hence the number required is greater than 34. Every number greater than 35 can always be written as 5m +7n for suitable positive integral values of m and n. 110 Sample Questions and Solutions ll For example take the number 103. A multiple of 7 ending in 3 being 63, we can write 103 as 7x9+5x8. Similarly, 245 = 210 +35 = 7x 30+5x7 or 5x42+7x5. For no integers m,n > 0. 35 can be written as 5m+7n. For m =1,2,3,4. We find 5m. is 5,10,15,20,15 none of which is a multiple of 7. For m = 2, n has to be 0 and not a positive integer. Thus 35 is the largest positive integer of the required form. «. Answer is (B) 3. The last digit in the finite decimal representation of the number (1) is 5 (A) 2 (B) 4 (0) 6 (D)8 Screening Test 2004 Solution: We have 1" sooo, (5) = 2 pe Now 24 = 16 and 27 = 16 x 16 x -+-501 factors. It is easily seen that two numbers ending in 6 when multiplied give another number ending in 6. Thus the last digit in 22004 ig 6, Hence in 2704/1004 the last digit of the decimal representation will also be 6. ., Answer is (C) 4, Three people each think of a number, which is the product of two different primes. The product of the three numbers which are thought of is (A) 120 (B) 12100 (©) 240 (D) 3000 Screening Test 2004 Solution: The product of the three numbers is clearly a composite number which can be expressed as the product of 6 primes of which at least two must be distinct. Using prime 112 Mathematics Talent Tests - Sub-Junior Level factorisation 120=29x3x5, 12100 = 2? x 5? x 117, 240 = 24 x 3x5, 3000 = 23 x 3 x 53 We find that only 12100 is expressable as the product of six primes while the others cannot be expressed as product of six primes. The numbers thought by the three persons are 2x5=10, 5x 11=55 and 11x 2=22. . Answer is (C) PART-II 1. Show that if n is a positive integer, then n° —n is always divisible by 3. B—n n(n? — 1) n(n+1)(n—1) —— ‘Three consecutive numbers or (n=1)xnx (n+) ‘Three consecutive numbers Since n?—n is a product of three successive integers, one of these integers will be a multiple of 3 and hence n?—n should be divisible by 3. 2. Show that among all the prime numbers there is no greatest prime. The list of prime numbers goes like this: 2,3,5, 7,11, 13,17,-+- ete. Suppose that there is a largest prime P. Sample Questions and Solutions 113 Consider the number Qa(2xX3x5x7X1LX--+x P)F1 [This is obtained by multiplying all the primes together and adding 1]. Clearly Q > P. Therefore, Q cannot be prime, by our supposjtion. If it is not prime then it must be divisible by some prime. But no such prime exists in our list since they leave remainder 1 when divided into Q. Therefore, our supposition must be wrong. Therefore, there is no largest prime number. Note: This method is known as ‘Method of Contradiction’. 3. Find the smallest number which when divided by 10 leaves a remainder of 9, when divided by 9 leaves a remainder of 8, when divided by 8 leaves a remainder of 7,--- etc. down to where, when divided by 2 it leaves a remainder 1. Let N be the required number. N 10a, +9 = 9a, +8 = 2a, +1 ¢. N+1= 10(ag +1) = 9(ag +1) = +++ 2(a, +1) ie N+1 has factors 2,3,4,---10 whose L.C.M is = 23 x 3? x 5 x 7 = 2520. Therefore, N is 2519. ld Mathematics Talent Tests - Sub-Junior Level Note: ‘The other numbers having the property are 2520 x 2-1 2520 x 3-1 2520 x 4-1 etc 4. ‘The sum of two natural numbers is 100. Show that their product cannot be greater than 2500. Let a,b be the numbers, a+b= 100 ‘Therefore ab = a(100- a) = 100a-a? = 2500 — 2500 + 1000 — a? = 2500 — (a? — 100a + 2500) = 2500- (a — 50)? a this is a pve quanting = 2500 — a positive quantity Therefore, ab < 2500. 5. The sum of two numbers is a constant. Prove that the product is the maximum when the numbers are equal. We know that (x+y)? — («-y)? = 4ey ‘The sum of the numbers ,y is a constant (say) k aty=k (1) Q) 3 B-(e-y)? =4ey @) In (2) the RHS is 4cy. If 4zy is maximum, clearly the product zy is maximum, For that, the LHS should be maximum. k isa constant. Thus a minimum quantity should be subtracted from k? so that k? — (x —y)? is maximum. 2 Sample Questions and Solutions 115 But (a —y)? is always positive. The minimum value of the positive quantity is zero, Thus (w©-y)?=0 »2-y=0 >r=y. ‘Thus the numbers are equal.[Now read Qn.4 again]. . A isa set of 2004 positive integers. Show that there is a pair of elements in A whose difference is divisible by 2003. Final Test 2004 Solution: When we divide a number by 2003 there are 2003 possible remainders, namely, 0,1,2,3,...,2001,2002. Now, since A contains 2004 positive integers, there are at least two numbers in A which leave the same remainder when divided by 2003. Call them a, and az. Then the difference Ja; — a3| is divisible by 2003 . . a > b > c are three consecutive numbers. Show that FAB +. Let a=n+1,b=n,c=n-—1 be the consecutive natural numbers. Also given a > b> c. If possible let a® = 0° + c3 () Therefore, (n + 1)3 =n? +(n—1)* > 2 = n?(n—6) after implitieation. ‘The right side n?(n — 6) is positive if and only if n > 6. In that case n?(n — 6) > 36. This cannot be so because 2 = n?(n—6). So, our assumption (1) is wrong. . Is the statement “If p ‘and p? +2 are primes then p? +2 is also a prime” true or false? Give reasons for your answer. Final Test 2004 Solution: Suppose p and p? +2 are primes. One such Prime number is p = 3. If p # 3, then p = 3k +1 and 6 Mathematics Talent Tests - Sub-Junior Level p?+2 = (3k +1)? +2 = 9k? + 6k -+3 which is always a multiple of 3 and it is always greater than 3. for all integral values of k. Hence for p #3 we see that p? +2 is never a prime number. So, p and p? +2 are primes if and only if p= 3. In this case p? +2 = 29 is also a prime. Thus the statement “If p and p? +2 are primes then p® +2 is also a prime” is always true! Find the natural number n. such that 2!9 + 219 + 2" is a perfect square of an integer. Final Test 2002 Solution: Let 2842! 42" = y? 2(8+1)4+22 = ¥? (Bx3Pr+a = x r= y— 96 = (y+96)(y — 96) , Each of (y +96) and (y—96) should be a power of 2. Since they differ by 192, we have y+96 = 256=28 y-96 = 64=2° ©. The only solution is n= 8+6 = 14. Aliter: Let a? = 21342104 on 210(1 +23 + gn-10) = 210g 4 gn-10) Wl 9+2"-10 is a perfect square, say, y?. This means 3, 20"-10/2, y are Pythagorean triads. Hence 2°-10)/2 = 92 = 4,y = 5, o Bf =2 or n= 14. Sample Questions and Solutions 117 10. If the square of any odd natural number is divided by 8, show n, that the remainder will always be 1. Any odd natural number is of the form 2n +1. Its square = (2n + 1)? = 4n?+4n+1=4n(n+1)+1 = (4x even number) + 1 [Because n(n+1) is the product of two consecutive numbers, it should be even] = (a multiple of 8) +1 ‘Therefore, when we divide this, remainder is 1. A number when added to either 100 or to 168 yields a perfect square natural number. What are these square numbers? Let’ x be the number which we start with. Let the square numbers be a? and 6? (and let a? > b*) Therefore, a? — 6? = (a + 168) — (x + 100) = 68 ‘Therefore, (a + 6)(a — 6) = 68 = (17 x 4) or (34 x 2) If (a+6)(a—b) =17x4 a+b6=17 and a—b=4 and so 2a = 1 which means a=10.5 This is not possible since a cannot be a fraction. If (a+ b)(a—b) = 34 x2 a+6 = 34 and a—b = 2 and 0 2a = 36 which means a=18;b=16 : a=18>07=324 b= 16 = b? = 256 The required numbers are 324, 256. Note : The starting number is 256 - 100 = 156. 118 Mathematica Talent Teats ~ Sub-Junior Level 12, Find the first positive integer whose square ends in three 4's. ‘Tho first such number is 444, It is not a square because 21? = 441 and 22? = 484. ‘The noxt such square number should be a four-digit number ending in 444, Let this number be 2. Then & > 900(= 307) Therefore, it could be 32? or 38? or 42? or 48 etc. 32? = 1024, This does not satisfy the condition that should end with triple 4’s. Next, 38? = 1444. This clearly satisfies with our requirements. [Note: You cannot have any square number ending with four 4’s or four 6's, ete.] 13, Let P denote the product of first n prime numbers(with n> 2), For what values of n we have 1, P-1 isa perfect square 2. P+1 isa perfect square Final Test 2004 Solution: Since n > 2, we have P as a multiple of 3. So P—1 isof the form 3k-+2 for some positive integer &. But any perfect square is either a multiple of 3 or of the form 3m+1. Thus P—1 is never a perfect square for n > 2. Now, P +1 is an odd integer and hence if P+1 = 0? then 6 must be an odd number. Let 6 = 2m+1. Then P = #-1 = 4m?+4m. This means that 4 divides P=2x3x5x7x 11L..(m factors) which is impossible. ‘The answer to the question is that there is no such n > 2. CHAPTER - X DATA HANDLING PART-I 1. The number halfway between } and 5 is A)y B) y o% D) ‘ t Required number = ©. Answer is (2). 2. Five years ago, the average age of A,B,C and D was 46 years, With E joining them now, the average age of all the five is 49 years. The present age of E is A) 40 B) 45 c) 42 D) 48 5 years ago, A+ B+O+D = (45 x 4) = 180 years Now A+B+C+D =(180+5 x 4) = 200 years Also A+B+O+D+B = (49x 5) = 245 years Therefore, Age of B now = (245 — 200) = 45 years .:. Answer is (B). 3. One hundred and twenty students take an examination which is marked out of 100 (with no fractional marks), No three students are awarded the same mark. What is the smallest possible number of pairs of students who are awarded the same mark? (A) 9 (B) 10 (C) 19 (D) 20 Screening Test 2004 119 120 Mathematics Talent Tests - Sub-Junior Level Solution: If every mark from 0 to 100 is given one each to 101 students, the remaining 19 students will have to be awarded marks already given. As no three students score the same mark, the minimum possible pairs of students getting equal marks is 19. . Answer is (D) 4. The median of the numbers —4,2,0,-6,5,8 is A)l B)2 co D) -4 Arranging in ascending order ~6,—4,0,2,5,8 ‘Therefore, median = Middlemost item (0+2) 2 PART-II 1. Appu has 6 marks (all integers) on his Report Card. i) The mean of the 6 marks is 74. ii) The mode of the 6 marks is 76. iii) The median of the 6 marks is 76. iv) The lowest mark is 50. v) The highest mari is 94. vi) Only one mark appears twice and no mark appears more than twice. Find the number of possibilities for his second lowest mark. Final Test 2002 Solution: (ii) & (vi) + Two of the marks are 76,76. (iv) & (v) => Four of the marks are 50,76,76,94. Sample Questions and Solutions 121 We have an even number of marks (namely six) and so (iii) = 76 should occupy two places, namely the 3"¢ and the 4th, when arranged in ascending order. Let P be the second lowest and Q be the second highest among the six marks. ‘That is, the marks arranged in ascending order are 50, P, 76, 76, Q, 94 This shows that P can be any natural number from 51 to 75 and Q can be any natural number from 77 to 93. (a) Now (i) = SEP HTOATOLGION _ 74 gy PHOI6 _ 74 and so Q = 148 - P (6) By (a), we find Q has 17 possibilities (namely from 77 to 93). When Q = 77, by (8) we have the largest P(= 71); and when Q= 93, we have the smallest P(= 55). Thus P has 17 possibilities.(from 55 to 71). This means that there are 17 possibilities for the second smallest mark (=P). 2. Vasanth read a eleven page article in an Encyclopedia. The sum of the age numbers he read is 1 less than 2003. What are the eleven page numbers? Fina hea Solution: Let the numbers be a1, 42,43, a4, 05, -++ dn, ag is the average of a; and ay. *, ag = (ay +a) ag is the average of az and aio. *. ag = 4(ap + ayo) ag is the average of a3 and ag. *. ag = (a3 +9) is the average of ay and a3. ag = Hay +a5) a6 ag is the average of as and a7. +, ag = 4(a5 +a7). 122 Mathematics Talent Tests - Sub-Junior Level «5 times ag = 3(a1+a11+02+a10+a3-+a9+a4-+ag-+a5-+a7) -. 10 times ag = (a)+a2+a3+a4+a5+a7+ag+a9+a10+011) <. 10 times ag = (a, +02 +43 +.04 +45 +.a5 +07 + ag +a9+ ayo + an) 11 x ag = 2002 (given) +, ag = 182. .. The page numbers are: 177,174,179,180,181,182,183, 184,185,186,187. Solution 2: The page numbers beginning with a, are a,q; + 1,0; + 2,0; + 3,0) +4,-++ ,a, +10. Adding, 11 x a, +55 = 2002. », 11a; = 1947. a, =177. .. The numbers are 177, 178,--- ,187.

Potrebbero piacerti anche